Illustrated Questions in Orthodontics [2 ed.] 9780191024191, 9780198714828

Illustrated Questions in Orthodontics takes a problem-based approach to orthodontics, offering a unique resource for und

273 221 69MB

English Pages 177 Year 2015

Report DMCA / Copyright

DOWNLOAD FILE

Polecaj historie

Illustrated Questions in Orthodontics [2 ed.]
 9780191024191, 9780198714828

Citation preview

Illustrated Questions in Orthodontics

Illustrated Questions in Orthodontics Claire Nightingale Consultant Orthodontist, Watford General Hospital and Specialist Orthodontist, Queen’s Gate Orthodontics, London

Jonathan Sandy Professor of Orthodontics, School of Oral and Dental Sciences, University of Bristol

1

1 Great Clarendon Street, Oxford, OX2 6DP, United Kingdom Oxford University Press is a department of the University of Oxford. It furthers the University’s objective of excellence in research, scholarship, and education by publishing worldwide. Oxford is a registered trade mark of Oxford University Press in the UK and in certain other countries © Oxford University Press 205 The moral rights of the author[s]‌have been asserted First edition published in 2000 © Reed Educational and Professional Publishing Ltd Impression:  All rights reserved. No part of this publication may be reproduced, stored in a retrieval system, or transmitted, in any form or by any means, without the prior permission in writing of Oxford University Press, or as expressly permitted by law, by licence or under terms agreed with the appropriate reprographics rights organization. Enquiries concerning reproduction outside the scope of the above should be sent to the Rights Department, Oxford University Press, at the address above You must not circulate this work in any other form and you must impose this same condition on any acquirer Published in the United States of America by Oxford University Press 98 Madison Avenue, New York, NY 006, United States of America British Library Cataloguing in Publication Data Data available Library of Congress Control Number: 20495247 ISBN 978–0–9–87482–8 Printed in Great Britain by Ashford Colour Press Ltd, Gosport, Hampshire Oxford University Press makes no representation, express or implied, that the drug dosages in this book are correct. Readers must therefore always check the product information and clinical procedures with the most up-to-date published product information and data sheets provided by the manufacturers and the most recent codes of conduct and safety regulations. The authors and the publishers do not accept responsibility or legal liability for any errors in the text or for the misuse or misapplication of material in this work. Except where otherwise stated, drug dosages and recommendations are for the non-pregnant adult who is not breast-feeding Links to third party websites are provided by Oxford in good faith and for information only. Oxford disclaims any responsibility for the materials contained in any third party website referenced in this work.

Preface The predecessor to this book, Orthodontics:  Picture Test Atlas, published by Wright in 200, was a great success, receiving good reviews and an award from the Society of Authors. It was translated into several languages and exceeded its predicted volume of sales within one print run. In this second edition, much of the content has remained the same as the original, as the book is primarily focused on the principles of orthodontics rather than technique. However, significant developments have occurred within clinical orthodontics since the first edition, such as the use of self-ligating brackets, temporary orthodontic anchorage, treatment with clear aligner systems, and bespoke lingual appliances, and this second edition has been updated to include questions on these. Only some appliances, such as the Roberts Retractor and removable appliances for canine retraction, which are now of historical interest, have been removed. We hope this new edition will stand the test of time and remain in prolonged circulation, bringing the benefits we intended to those with an interest in understanding the art and science of the practice of orthodontics. Claire Nightingale, London, 204 Jonathan Sandy, Bristol, 204

Acknowledgements We would very much like to thank the people who have consented for their or their child’s facial images to appear in this book. We are particularly grateful to Miles Maidment, Medical Photographer at West Herts NHS Trust, who scanned our original transparencies and endlessly reproduced new images, efficiently and enthusiastically. Finally, we express our gratitude to our long-suffering spouses, Stuart Nightingale and Jackie Sandy, who were accompanied by this edition, in constantly edited draft form, on too many mini-breaks.

Contents About the authors  viii Abbreviations  ix

Section 1 Questions  2 3 4 5 6 7 8 9 0

Ideal occlusion Examination and diagnosis Treatment planning Pathology Removable appliances Functional appliances Fixed appliances Anchorage Problems Retention

2 4 34 46 58 66 72 84 90 98

Section 2 Answers  2 3 4 5 6 7 8 9 0

Ideal occlusion Examination and diagnosis Treatment planning Pathology Removable appliances Functional appliances Fixed appliances Anchorage Problems Retention

Index  63

04 06 22 28 34 42 46 52 56 60

About the authors Claire Nightingale MSc, BDS (Hons), FDS (Orth) RCS, MOrth RCS (Edin), FDS RCS (Eng) Claire is a Consultant Orthodontist based at the Department of Orthodontics, Watford General Hospital, where she is also Departmental Lead Clinician. She also runs a private orthodontic practice in London. Jonathan Sandy BDS (Hons), DOrth, MSc, FDS RCS (Edin), FDS RCS (Eng), MOrth, PhD, FFD (Irl), FGDP (UK), FHEA, FMed Sci, Jonathan is Professor of Orthodontics, School of Oral and Dental Sciences, University of Bristol. Head of the School 2007–4.

Abbreviations AC APo CPAP DHC DPT EOT IOTN MIH MOA MPO PAR RME TAD URA

aesthetic component ‘A’ point to pogonion continuous positive airway pressure dental health component dental panoramic tomogram extraoral traction Index of Treatment Need molar incisor hypomineralization medium opening activator mutually protected occlusion Peer Assessment Rating rapid maxillary expansion temporary anchorage device upper removable appliance

Section one

Questions  Ideal occlusion  2 2 Examination and diagnosis  4 3 Treatment planning  34 4 Pathology  46 5 Removable appliances  58 6 Functional appliances  66 7 Fixed appliances  72 8 Anchorage  84 9 Problems  90 0 Retention  98

CHAPTER ONE Ideal occlusion Question . A How would you classify this incisor relationship? B What are the other occlusal features you can see? C What are Andrew’s six keys to normal static occlusion? D What is the difference between an Angle’s and an Andrew’s class  I  molar relationship?

Figure .

Ideal occlusion  3

Question .2 A B C D

What do you see here? What is the name given to this type of functional occlusal relationship? What are the features of this type of dynamic occlusion? What is the other main type of dynamic occlusion?

Figure .2

CHAPTER TWO Examination and diagnosis Question 2. A What would you assess during an extraoral orthodontic examination? B What would you assess during an intraoral orthodontic examination? C What adjunctive procedures would aid diagnosis?

Examination and diagnosis  5

Question 2.2 A What is the anteroposterior skeletal relationship shown here? B How are the maxilla and mandible related in this type of skeletal relationship? C The cephalometric value ANB in the Eastman Analysis indicates an anteroposterior skeletal relationship. What is the likely value of the angle ANB in the skeletal relationship shown? D It is important to assess the vertical skeletal pattern as well as the anteroposterior skeletal relationship. What is a normal vertical skeletal relationship and what is the normal value of the Frankfort–mandibular planes angle?

Figure 2.2

6 

Illustrated Questions in Orthodontics

Question 2.3 A Classify this malocclusion. B What methods are used to relieve crowding? C Which feature of this malocclusion suggests that this patient needs treatment with fixed appliances?

Figure 2.3

Examination and diagnosis  7

Question 2.4 A What type of malocclusion is seen here in this -year-old girl? B A number of aetiological factors can cause this malocclusion. What factors would you consider? C What treatment options would you consider?

Figure 2.4a

Figure 2.4b

8 

Illustrated Questions in Orthodontics

Question 2.5 A Assess the extraoral features seen here. B Describe and classify the malocclusion. C If this case is amenable to orthodontic correction, which dimension in particular must be carefully controlled during treatment and why? D Will the result be stable after orthodontic treatment? E What alternative treatment option could be considered?

Figure 2.5a

Figure 2.5b

Examination and diagnosis  9

Question 2.6 A Assess the extraoral features of this patient. B Describe her occlusion and account for the anterior open bite. C She is 20 years old and greatly dislikes her profile and inability to incise properly. What treatment options would you discuss with her?

Figure 2.6a

Figure 2.6b

Figure 2.6c

Figure 2.6d

10 

Illustrated Questions in Orthodontics

Figure 2.6e

Examination and diagnosis  11

Question 2.7 A What dental and skeletal relationship do you see here? B Describe the key features of the soft tissue profile. C What is the principal cause of his increased overjet (9 mm) and what factors will influence post-treatment stability once the overjet has been reduced? D Why is lip competence important?

Figure 2.7b

Figure 2.7a

Figure 2.7c

12 

Illustrated Questions in Orthodontics

Question 2.8 A Describe the skeletal relationship shown here. B What value of the angle ANB is typical of this type of anteroposterior skeletal relationship? C The patient has a class II division 2 type incisor relationship. What are the skeletal and soft tissue features commonly seen with this malocclusion? D Which of her facial features masks the skeletal relationship?

Figure 2.8

Examination and diagnosis  13

Question 2.9 A Describe the buccal segment and incisor relationships. B Classify and define this malocclusion. C What are the dental features of this type of malocclusion? D Which principal feature of this malocclusion must be corrected to give the best chance of stability? E What is the cause of the upper incisor inclination?

Figure 2.9

14 

Illustrated Questions in Orthodontics

Question 2.0 A Assess the extraoral features of this -year-old girl. B Describe and classify her malocclusion. C What are the skeletal features commonly seen in people with this malocclusion? D This malocclusion is amenable to orthodontic correction. What are the aims of treatment? E What principal factor would limit the success of orthodontic treatment?

Figure 2.0a

Figure 2.0b

Examination and diagnosis  15

Question 2. A What do these photographs of this patient demonstrate and what is the clinical significance? B Why is the buccal segment relationship class  I  but the incisor relationship class III?

Figure 2.a

Figure 2.b

16 

Illustrated Questions in Orthodontics

Question 2.2 A B C D E

What type of skeletal relationship is this? What value of the angle ANB is typical of this type of skeletal relationship? How is the maxilla related to the mandible anteroposteriorly? What is the cause of the anterior open bite? Is this case likely to be amenable to orthodontic correction?

Figure 2.2a

Figure 2.2b

Examination and diagnosis  17

Question 2.3 A What skeletal relationship do you see here? B Describe the buccal segment relationship and the incisor relationship. C The facial profile suggests that the malocclusion is not as severe as the occlusion demonstrates. Explain this apparent anomaly.

Figure 2.3a

Figure 2.3b

18 

Illustrated Questions in Orthodontics

Question 2.4 A What do you see here? B Why is growth potential important in assessing this problem? C Why is it important to detect mandibular displacements on closure? D How would you manage a lateral mandibular displacement in a child in the mixed dentition?

Figure 2.4

Examination and diagnosis  19

Question 2.5 A B C D E

What type of malocclusion is shown here? In which ethnic groups is this problem more common? What is the aetiology of this malocclusion? What are the common motives for having orthodontic treatment? What factors influence the long-term stability of treated cases?

Figure 2.5a

Figure 2.5b

20 

Illustrated Questions in Orthodontics

Question 2.6 A B C D

When should the upper permanent canines (i) be palpable (ii) erupt? What feature of this maxillary arch causes concern? What investigations should you carry out? What treatment options would you consider?

Figure 2.6

Examination and diagnosis  21

Question 2.7 A B C D E

What has happened here? What is the incidence of this problem? Is this more common in males or females? What factors are associated with this condition? What are the risks associated with this problem?

Figure 2.7

22 

Illustrated Questions in Orthodontics

Question 2.8 A What do you see here? B What are the possible causes? C What will need to be monitored carefully during orthodontic treatment?

Figure 2.8

Examination and diagnosis  23

Question 2.9 A What do you see here? B What is the likely cause of this malocclusion? C What are the skeletal and dental features associated with this problem long term? D How would you manage this case in the first instance?

Figure 2.9

24 

Illustrated Questions in Orthodontics

Question 2.20 A Describe what you see here. B Is this a significant problem? C How would you manage this? D Design the appliance you would use to correct this problem. Teeth present are: 6EDC2 6EDC2

Figure 2.20

2CDE6 2CDE6

Examination and diagnosis  25

Question 2.2 A B C D

What has happened here? How common is this problem and is it more frequent in females or males? What factors would influence treatment? What are the aetiological factors contributing to this problem?

Figure 2.2

26 

Illustrated Questions in Orthodontics

Question 2.22 A What can you see here? B How should the space be maintained before orthodontic treatment? C How can aesthetics be managed during orthodontic treatment? D What options would you discuss with the patient for restorative management after orthodontic treatment?

Figure 2.22

Examination and diagnosis  27

Question 2.23 A B C D

Describe what you see here. What has happened to the deciduous molars? What is the prognosis for the permanent dentition? What are the possible causes of the anterior open bite?

Figure 2.23

28 

Illustrated Questions in Orthodontics

Question 2.24 A B C D

Describe what you see here. How would you investigate this problem? What treatment options would you consider? What aetiological factors may have caused this problem?

Figure 2.24

Examination and diagnosis  29

Question 2.25 A What condition does this radiograph demonstrate? B This patient is 9 years old. Which teeth are missing? C List the most commonly missing teeth in the Caucasian population and give the incidence rate for each. D What is the most commonly missing tooth in the Japanese population? E Name three syndromes associated with missing teeth.

Figure 2.25

30 

Illustrated Questions in Orthodontics

Question 2.26 A What problem is shown here? B What is the incidence of this problem? C What treatment options would you discuss? D How can the teeth adjacent to the central incisors be modified for better aesthetics?

Figure 2.26

Examination and diagnosis  31

Question 2.27 A What do you see here? B What is the cause of this problem? C How should this be managed?

Figure 2.27a

Figure 2.27b

32 

Illustrated Questions in Orthodontics

Question 2.28 A Account for the difference between these two photographs. B Why is this important?

Figure 2.28a

Figure 2.28b

cHAPTER THREE Treatment planning Question 3. A The aims of orthodontic treatment are individual to each patient, depending on the presenting features of the malocclusion. However, some general aims of treatment are often referred to. What are these? B What is the importance of assessing the skeletal relationship in all three planes? C What is the relationship between the vertical dimension and the decision to extract teeth?

Treatment planning  35

Question 3.2 A Why is growth potential significant in the assessment of the skeletal relationship? B Which treatment aims are most helped by favourable growth? C When is the pubertal growth spurt in (i) girls and (ii) boys? D What is a backward mandibular growth rotation and what is its importance to: i the correction of a class II skeletal relationship? ii overbite management? iii treatment mechanics?

36 

Illustrated Questions in Orthodontics

Question 3.3 A What is the importance of establishing a normal overbite as shown here? B What is the relationship between the vertical skeletal dimension and overbite? C How does the overbite affect overjet management? D Why is overbite reduction easier in a growing patient than an adult? E List methods of overbite reduction.

Figure 3.3

Treatment planning  37

Question 3.4 A How many millimetres of crowding would be described as: (i) mild, (ii) moderate, and (iii) severe? How crowded is this arch? B A premolar width is approximately 7 mm and this is described as one unit. What buccal segment relationship will result at the end of orthodontic treatment following the removal of: (i) one unit in each quadrant, (ii) two units in the lower arch only, and (iii) two units in the upper arch only in a patient with a full complement of permanent teeth and a class I skeletal relationship? C What factors would you take into account when planning extractions?

Figure 3.4

38 

Illustrated Questions in Orthodontics

Question 3.5 A Following orthodontic diagnosis and establishment of treatment aims, what is the starting point for treatment planning? B Which dimensions should not be altered during treatment?

Treatment planning  39

Question 3.6 A At what stage of dental development is this child? B What is the most important diagnosis at this stage? C What would be the likely effect of early unilateral loss of a deciduous canine on the developing dentition? D What would be the likely effect of early unilateral loss of a deciduous second molar on the developing dentition? E What interceptive measures would you consider to manage a maxillary permanent canine that appears to be palatally positioned?

Figure 3.6

40 

Illustrated Questions in Orthodontics

Question 3.7 A What is a balancing extraction and when would you consider this? B What is a compensating extraction and when would you recommend it? C When would you consider providing a space maintainer?

Treatment planning  41

Question 3.8 A This is a lateral cephalometric radiograph. How should the patient be positioned when it is taken? B Why are these radiographs taken? C What steps should be taken to produce an optimum image?

Figure 3.8

42 

Illustrated Questions in Orthodontics

Question 3.9 This is a tracing of a cephalometric radiograph of a patient. The following values were obtained from the tracing: SNA = 8.5º SNB = 93.5º ANB = –2º Maxillary mandibular plane = 5º Overjet = –2 mm Upper incisors to maxillary plane = 33º Lower incisors to mandibular plane = 80º A Describe the skeletal relationship both vertically and anteroposteriorly. B Compare the incisor angulations to normal Caucasian values and account for the differences. C Describe the overjet. Is this what you would expect to find in a patient with this skeletal relationship? D Is the maxilla normally positioned? E Will this patient’s malocclusion be manageable by orthodontics alone?

Figure 3.9

Treatment planning  43

Question 3.0 The following values were obtained from this tracing of a lateral cephalometric radiograph of a patient: SNA = 8.5º SNB = 75.5º ANB = 6º Maxillary mandibular plane = 24.5º Overjet = 6 mm Upper incisors to maxillary plane = 99º Lower incisors to mandibular plane = 98º A Describe the incisor relationship. B Which jaw is likely to be the cause of the skeletal discrepancy and why? C Comment on the inclination of the incisors and describe what effect this has had on the overjet?

Figure 3.0

44 

Illustrated Questions in Orthodontics

Question 3. The PAR and IOTN Indices are well-established in orthodontics. A What do the acronyms stand for? B What is the difference between them? C How is the IOTN further divided?

CHAPTER FOUR Pathology Question 4. A B C D

What is this? How frequently does it occur? What causes it? Describe the mechanism and timing of palatal shelf closure.

Figure 4.

Pathology  47

Question 4.2 A Unilateral cleft lip and palate is the most common form of facial clefting. What are the other expressions of clefting in humans? B This photograph demonstrates a unilateral left-sided cleft lip repair. What is the typical surgical management of the repair of clefts? C Management of cleft lip and palate requires a team approach within specialist centres. Which specialists are ideally involved? D What is the role of the orthodontist in the management of cleft lip and palate and when would they be typically involved?

Figure 4.2

48 

Illustrated Questions in Orthodontics

Question 4.3 A This photograph demonstrates a patient who had a bilateral cleft lip and palate repaired as a neonate. How is the dentition affected in patients with clefting? B What are the major issues of concern in the dental management of these patients?

Figure 4.3

Pathology  49

Question 4.4 A B C D

What soft tissue abnormality is seen here? What effect can it have on the dentition? What treatment should be considered? What physical sign is a guide to prescribing surgery?

Figure 4.4

50 

Illustrated Questions in Orthodontics

Question 4.5 A B C D

What is this? Is this a common appearance? What is the incidence of this problem? What problems can arise?

Figure 4.5

Pathology  51

Question 4.6 A What abnormality in eruption is seen here? B How should this be managed? C What are the common causes of this problem?

Figure 4.6

52 

Illustrated Questions in Orthodontics

Question 4.7 A What do you see here? B What is the potential problem and how might it be managed? C What impact will this anomaly have on the anterior occlusion?

Figure 4.7a

Figure 4.7b

Pathology  53

Question 4.8 A B C D

What do you see here? How frequently does this occur? What treatment options exist? What impact will it have on the anterior occlusion?

Figure 4.8a

Figure 4.8b

54 

Illustrated Questions in Orthodontics

Question 4.9 A What do you see here and what is the name given to the condition affecting the lower right first molar? B What are the possible causes? C What symptoms may be associated with this condition? D How would you manage this problem? E When is the best time to extract first molars?

Figure 4.9

Pathology  55

Question 4.0 This is a histological section of a tooth, periodontal ligament, and bone. A Describe the activity at the bone/periodontal interface. B How vascular is the periodontal ligament? C In which direction is the tooth moving?

Figure 4.0

56 

Illustrated Questions in Orthodontics

Question 4. This is a compressed periodontal ligament. A Describe the appearance. B What is likely to happen once the force has been removed? C Why is this appearance undesirable during orthodontic tooth movement?

Figure 4.

Pathology  57

Question 4.2 This is a histological section demonstrating tooth movement. A Describe the bone/periodontal interface. B In which direction is the tooth moving? C What is the likely origin of the large multinucleate cells (osteoclasts)?

Figure 4.2

chapter five Removable appliances Question 5. A What is the fundamental difference between tooth movement achieved with removable or fixed appliances? B What controls the force delivered by removable appliances and what is considered to be the optimal force? C What are removable appliances used for in current orthodontic practice? D What are the advantages and disadvantages of removable appliances when compared to fixed appliances?

Removable appliances  59

Question 5.2 A It is useful to use the acronym ARAB as a memory aid when designing removable appliances. What does this stand for? B Design an appliance to push the upper incisors over the bite in the patient seen here. Teeth present are: 6EDCB 6EDC2 C Is retention necessary after treatment?

Figure 5.2

2CDE6 2CDE6

60 

Illustrated Questions in Orthodontics

Question 5.3 A What are the two functions of this appliance? B How often should the screw be turned and how much activation does this achieve? C What is the disadvantage of this method of expansion? D What other methods of upper arch expansion do you know? E What is the function of the posterior bite capping and of what should the patient and parent be warned? F How are the T springs activated and what alternative springs could be used? G Which feature of the malocclusion will be the most prone to relapse? H Draw a design for this appliance.

Figure 5.3

Removable appliances  61

Question 5.4 .

A B C D

What is the purpose of this appliance? What are the problems with its use? What is the purpose of the posterior capping? Why are tubes soldered to the bridge of the Adams cribs?

Figure 5.4

62 

Illustrated Questions in Orthodontics

Question 5.5 A Draw designs for the following upper removable appliances (URAs). Assume all permanent teeth, excluding third molars, are present: i an appliance designed to distalize the upper first molars in conjunction with headgear attached to molar bands ii a clip-over bite plane to be used with fixed appliances iii a space maintainer for a missing upper left central incisor iv an appliance designed to achieve unilateral crossbite correction of the upper right buccal segment v an appliance designed to distalize the upper left buccal segment. B All appliances used to distalize teeth share one common problem. What is this and how can it be monitored and controlled?

Removable appliances  63

Question 5.6 A B C D E F G

What is this appliance? What is it suitable for? What is the process for its manufacture and supply? What additions aid tooth movement? How much tooth movement can be achieved? What instructions would you give for its use? What adjunctive mechanics can you see?

Figure 5.6

64 

Illustrated Questions in Orthodontics

Question 5.7 A B C D E F

What is this appliance? What is it used for? How does it work? What steps are taken to construct it? What would you warn the patient about? What alternative therapy to a mandibular advancement splint is most commonly recommended to manage obstructive sleep apnoea?

Figure 5.7

chapter six Functional appliances Question 6. A What appliance is this? B How is it retained? C Which malocclusion is particularly suited to treatment with this type of appliance and why? D What are its limitations?

Figure 6.

Functional appliances  67

Question 6.2 A B C D E

What type of appliance is this? When would you use it? How does it work? What is the advantage of this appliance over other functional appliances? How would you attach headgear, if required? Why is the addition of extraoral traction (EOT) an advantage?

Figure 6.2

68 

Illustrated Questions in Orthodontics

Question 6.3 A B C D E

What type of appliance is this? What is its country of origin? How does it work? For which stage of dental development is this appliance particularly useful? Describe possible alternative treatment options for the management of a developing class III malocclusion.

Figure 6.3

Functional appliances  69

Question 6.4 A What is this? B When might it be useful? C How is it constructed and retained?

Figure 6.4

70 

Illustrated Questions in Orthodontics

Question 6.5 A B C D E

What is this appliance? How is it different from the other functional appliances shown? What is the advantage of this over the others? What is the disadvantage? Why does this patient have a reverse overjet?

Figure 6.5

chapter seven Fixed appliances Question 7. A What appliance is this? B How can you distinguish between the straightwire and standard edgewise appliances? C What are the advantages of the straightwire appliance over the standard edgewise appliance? D What components are used for a fixed appliance and how are they attached? E What type of elastic wear is this?

Figure 7.

Fixed appliances  73

Question 7.2 A What type of fixed appliance is this and which technique is its predecessor? B How does its mode of action differ from the straightwire appliance? C What should be achieved in each stage of treatment? D What is the mechanical driving force used in this system and how are its unwanted side-effects controlled?

Figure 7.2

74 

Illustrated Questions in Orthodontics

Question 7.3 A What can be seen here? B What are the advantages and disadvantages of this type of bracket? C How are the disadvantages being overcome?

Figure 7.3

Fixed appliances  75

Question 7.4 A B C D E

What type of brackets are these? How do they differ from conventional brackets? What other types of self-ligating brackets exist? What are the advantages and disadvantages of these types of bracket? When might they be used?

Figure 7.4

76 

Illustrated Questions in Orthodontics

Question 7.5 A B C D

What type of orthodontic technique and appliance is this? What are the advantages and disadvantages of this technique? What are the stages for construction and fitting of this appliance? What instructions would you give the patient on fitting?

Figure 7.5

Fixed appliances  77

Question 7.6 A What is happening here? B How is the appliance activated?

Figure 7.6

78 

Illustrated Questions in Orthodontics

Question 7.7 A What are the desirable properties of an orthodontic wire? B What effect does increasing the cross-section of a wire have on its properties? C What effect does increasing the length of a wire have on the forces being applied to teeth?

Fixed appliances  79

Question 7.8 A B C D E

What type of appliance is this? What is it used for? What is the normal amount of activation? What problems might be expected? What alternative appliances could be used?

Figure 7.8

80 

Illustrated Questions in Orthodontics

Question 7.9 A What is this appliance called? B How often should the screw be turned and when should the patient be reviewed after fitting? C Of what should the patient be warned? D What is the difference between the expansion achieved using this appliance compared to other appliances?

Figure 7.9

Fixed appliances  81

Question 7.0 A What is this and what is it trying to achieve? B Why have the upper first molars rotated? C What alternative approach could be used?

Figure 7.0

82 

Illustrated Questions in Orthodontics

Question 7. A What is this palatal appliance called and what is its purpose? B How is it constructed?

Figure 7.

chapter eight Anchorage Question 8. A B C D E

What appliance is this? What is its function and when is its use indicated? Describe the clinical steps in its fabrication. What problem might arise during transit to the laboratory? What alternative appliance might be considered?

Figure 8.

Anchorage  85

Question 8.2 A B C D E

What type of appliance is this? When would you use it? What are the component parts of the system shown here? What force would you apply and for how long? What precautions must be taken if this appliance is used?

Figure 8.2

86 

Illustrated Questions in Orthodontics

Question 8.3 A Describe this buccal segment relationship. B Give a possible explanation for this appearance. C The patient wishes to have comprehensive orthodontic treatment with full reduction of his 8 mm overjet. He has a class I skeletal relationship of normal vertical dimension with competent lips. What options are available for him?

Figure 8.3

Anchorage  87

Question 8.4 A What appliance is this? B What is it used for? C What are the problems which patients may experience in wearing the appliance? D To what is it attached intraorally?

Figure 8.4

88 

Illustrated Questions in Orthodontics

Question 8.5 A B C D

What is happening to this patient? What must be considered when the implant is placed? How soon can the implant be loaded after placement? What other uses can implants have?

Figure 8.5

Anchorage  89

Question 8.6 A What problem can you identify here? B How is the patient being managed?

Figure 8.6

chapter nine Problems Question 9. A B C D E

What has happened during treatment? What are the aetiological factors? Is this a common problem? How might this be avoided during treatment? What treatment is available?

Figure 9.

Problems  91

Question 9.2 A What do you see in the first picture? B What has happened during orthodontic treatment? C How soon should traumatically intruded teeth be extruded?

Figure 9.2a

Figure 9.2b

92 

Illustrated Questions in Orthodontics

Question 9.3 A What can you see here? B What possible aetiological factors would you consider? C How would you manage this problem?

Figure 9.3

Problems  93

Question 9.4 A B C D

What has happened during orthodontic treatment? Is this a common problem? What are the risk factors associated with this problem? How can this problem be minimized?

Figure 9.4

94 

Illustrated Questions in Orthodontics

Question 9.5 A What do you see here and what is the cause? B How would you manage this problem?

Figure 9.5

Problems  95

Question 9.6 A What do you see here and what is the cause? B How would you manage this problem?

Figure 9.6

96 

Illustrated Questions in Orthodontics

Question 9.7 A What do you see here and what is the cause? B How would you manage this problem?

Figure 9.7

Problems  97

Question 9.8 A What do you see here and what is the cause? B How would you manage this problem?

Figure 9.8

chapter ten Retention Question 0. A B C D

What appliances are these? What are they for and what regime for use would you prescribe? Is relapse common after orthodontic treatment? Outline some of the causes. What features of the dentition, pretreatment, are most prone to relapse and how can this relapse be minimized?

Figure 0.

Retention  99

Question 0.2 A What is this? B What are the advantages and disadvantages of this type of retainer compared to a removable acrylic retainer?

Figure 0.2

100 

Illustrated Questions in Orthodontics

Question 0.3 A B C D E F

What appliance is attached to the lingual surfaces of the teeth? What is it for and what situations might indicate its use? What problems could be encountered? What is the clinical procedure for making it? What alternative appliances exist? What is unusual about this patient’s dentition?

Figure 0.3

Retention  101

Question 0.4 A What do you see here? B What are the advantages and disadvantages? C Why is the fixed retainer attached to the premolars?

Figure 0.4

Section two

Answers  Ideal occlusion  04 2 Examination and diagnosis  06 3 Treatment planning  22 4 Pathology  28 5 Removable appliances  34 6 Functional appliances  42 7 Fixed appliances  46 8 Anchorage  52 9 Problems  56 0 Retention  60

chapter one Ideal occlusion Answer . A Incisor relationship is classified as class  I, II, or III, depending where the lower incisors occlude relative to the cingulum plateau of the upper central incisors. This is a class I incisor relationship where the lower incisors occlude with or lie directly below the cingulum plateau of the upper central incisors. The overjet usually measures 2–4 mm. B The cusp of the upper canine occludes in the embrasure between the lower canine and first premolar and this is called a class  I  canine relationship. The mesiobuccal cusp of the upper first molar occludes with the mesiobuccal groove of the lower first molar, which is an Angle’s class I molar relationship. There are no crossbites or scissorbites as the upper buccal segment teeth occlude just buccally to the opposing mandibular buccal teeth. Additionally, on maximum intercuspation, the mandibular condyles must be correctly seated in the articular fossae. C Andrew’s six keys to occlusion are based on his assessment of the study casts of 20 non-orthodontically created class I occlusions. He concluded that the six dental features common to all were: i correct molar relationship ii correct crown angulation iii correct crown inclination iv tight interproximal contacts v absence of rotations vi a flat curve of Spee.

Additionally, tooth size must be correctly proportioned to achieve a perfect class I occlusion. Most of these features are visible in this photograph. D An Angle’s class I molar relationship is such that the mesiobuccal cusp of the upper first molar occludes in the mesiobuccal groove of the lower first molar. To satisfy an Andrew’s class I molar relationship, the distal cusp of the upper first molar occludes with the mesial marginal ridge of the lower second molar, i.e. supra class  I  Angle relationship.

Answer .2 A This photograph demonstrates canine guidance during lateral mandibular excursion. B This functional occlusal relationship is called the mutually protected occlusion (MPO) or the canine guided occlusion.

Ideal occlusion  105 C The MPO is so called because the posterior teeth protect the anterior teeth in centric occlusion, whilst during dynamic movements, the anterior teeth protect the posterior teeth. Therefore, there is minimal load placed on the anterior teeth at maximum intercuspation. Incisal guidance is seen on protrusion and canine guidance achieved on lateral excursion. There is an immediate and gentle disclusion of the posterior teeth without occlusal interference during such movements. D The other main type of dynamic occlusion is called group function. In this scheme, all the buccal segment teeth on the working side, as opposed to just the canine teeth, guide the lateral excursive movements. The non-working side teeth should not contact and, therefore, are said to be free of interference.

chapter two Examination and diagnosis Answer 2. A An extraoral orthodontic examination should include: i the skeletal relationship in all three planes of space, i.e. anteroposterior, vertical, and transverse ii the soft tissue relationship and behaviour, especially the upper incisor/lower lip relationship, e.g. are the upper incisors sitting anterior to or on the lower lip (described as a lip trap); when the lips are relaxed, are they together (competent) or apart (incompetent)? iii the upper incisor/upper lip relationship, both dynamically and at rest, to assess the amount of upper incisor show and vertical maxillary excess (‘gummy smile’) iv the dental centrelines with respect to the facial midline, which will help identify facial asymmetry and mandibular displacement on closure, i.e if the lower dental centreline is not coincident with the upper dental centreline, in the absence of crowding, look for an underlying mandibular asymmetry or a mandibular displacement on closure. B An intraoral orthodontic examination should include: i tooth presence and quality ii oral hygiene and gingival condition iii lower labial segment crowding/spacing and tooth position (inclination, angulation, and rotation) iv upper labial segment crowding/spacing and tooth position v buccal segment crowding/spacing and tooth position vi anterior occlusion, i.e. overbite, overjet, and upper and lower centreline coincidence vii posterior occlusion, i.e. canine and molar relationship, interdigitation, and transverse relationships viii dynamic occlusion. C Adjunctive procedures which may aid diagnosis include:

i Radiographs

Usually a dental panoramic tomogram (DPT) to assess dental development and to detect gross pathology is required. An upper anterior occlusal view to aid localization of impacted teeth may be required if the DPT is unclear and impacted teeth are suspected. 3D imaging (e.g. cone beam tomography) is a very useful method of viewing impacted teeth, particularly assessing their relationship with adjacent structures such as other teeth and the inferior dental nerve. However, the extra radiation dosage must be justified compared to conventional radiography for any additional diagnostic yield. A  lateral cephalometric radiograph

Examination and diagnosis  107 to assess skeletal relationship and tooth inclination is required for patients with a skeletal discrepancy. Finally, long cone periapical views may be required to assess root anatomy if the DPT is suggestive of short or damaged roots or unusual root anatomy.

ii Study models

Conventionally, the bases of orthodontic study models are trimmed so that they can stand on any edge and represent the patient’s occlusion. Articulated study models, mounted using a face bow transfer, provide a truer representation of the patient’s occlusion with reference to the orientation of the functional occlusal plane, but are rarely used except for orthognathic patients.



Extraoral and intraoral photographs are desirable to record facial appearance, the standard of oral hygiene, and any enamel defects pretreatment.

iii Photographs

Answer 2.2 A B C D

This patient has a class I skeletal relationship. The maxilla is positioned slightly ahead of the mandible in a class I skeletal relationship. A class I skeletal relationship has an ANB value of 2–4°. The skeletal vertical dimension is assessed with the patient in profile. It consists of both linear and angular measurements. Classically, the face is considered as having three equally sized proportions, although this is not precise. These facial thirds are called the upper facial third (from the top of the forehead to soft tissue nasion), the midfacial third (soft tissue nasion to the base of the nose, a point called subnasale), and the lower facial third (subnasale to the base of the chin, a point called menton). The midfacial and the lower facial thirds are commonly considered together when they are referred to as the upper anterior and the lower anterior face height, respectively. Collectively, they form the total anterior face height. A person with normal facial vertical dimension has a lower anterior face height that is 55% of the total anterior face height. The Frankfort plane connects the infraorbital rim (orbitale) to the most superior point on the external auditory meatus (porion). The lower border of the mandible forms the mandibular plane. The intersection of these two planes forms the Frankfort–mandibular planes angle that typically has a value of 27° +/–5°.

Answer 2.3 A This is a class I occlusion, as the buccal segment teeth are class I. The canine relationship is not fully established as the upper canine has not completely erupted, but it appears likely to be class . However, the incisors are mildly crowded and the overjet and overbite are reduced, which suggests that the incisor relationship is slightly class III. This latter incisor relationship is one where the lower incisors occlude anterior to the cingulum plateau of the upper central incisors, whereas in class , the lower incisors occlude on the cingulum plateau of the upper incisors. B Space to relieve crowding can be created in four ways by: i dental extractions, the most common method of creating space ii distalization of the upper buccal segment teeth iii interdental enamel reduction, which is of use only for mildly crowded cases

108 

Illustrated Questions in Orthodontics



iv maxillary expansion. This can be considered for those cases with palatally inclined buccal segment teeth and those with a narrow maxilla, in whom rapid maxillary expansion, possibly supplemented by surgery, could be considered; also those with a unilateral crossbite resulting from a mandibular displacement on closure into centric occlusion. v incisor proclination. This method should only be considered if the incisors have been retroclined due to a habit, such as thumb sucking, or are trapped by the upper incisors, such as a class II/2 incisor relationship. In other circumstances, incisor proclination must be adopted very cautiously as proclining lower incisors, in particular, is an unstable procedure and can result in gingival recession if the lower incisors are proclined through the lower labial plate. All skeletal, soft tissue, especially upper facial profile, and dental features of the presenting malocclusion must be taken into consideration when deciding which method to use. C The upper and lower canines are distally angulated and their roots need to be moved distally using a fixed appliance. If the first premolars were extracted without fixed appliances being used, it is likely that the canine crowns would tip even further distally if left untreated, thereby worsening the initial tooth position. Additionally, the roots of the upper lateral incisors are palatally positioned and the crowns slightly rotated. This patient was managed with the removal of all second premolars and upper and lower fixed appliances.

Answer 2.4 A This patient is in the late mixed dentition and has a class II division  incisor relationship. This is determined by the lower incisors which occlude palatally to the cingulum plateau of the upper incisors, whilst the upper incisors are proclined or of average inclination with an increased overjet. There is a moderate–severe class II skeletal relationship with a reduced vertical dimension. B This type of malocclusion may be due to skeletal factors, such as mandibular retrognathia or maxillary prognathism, soft tissue factors, such as lower lip trapping or lip incompetence (lips not held together at rest), or habits, such as persistent thumb sucking. C This patient presents with well-aligned, spaced arches and should be treated on a non-extraction basis. This case is very suitable for functional appliance therapy, since the patient still has growth potential and is in the mixed dentition. Therefore, overjet reduction can be initiated whilst the permanent dentition is still being established. Additionally, this case is not crowded and may not need fixed appliance therapy after initial treatment has successfully reduced the overjet and overbite. Fixed appliances, or an upper removable appliance, with headgear, could also be successful, although retraction of the upper incisors should be avoided to prevent a detrimental effect on her upper facial profile. The upper incisors support the upper lip. Ideally, mandibular growth will give the best facial profile outcome in this case. Whichever method is chosen, it is clear that all orthodontic treatment relies on patient co-operation for success.

Answer 2.5 A This patient has a mild class II skeletal relationship, due to mandibular retrognathia, with an increased Frankfort–mandibular planes angle and an increased lower anterior facial height. B The buccal segment relationship is ¾ unit class II, the overjet is increased, and there is an anterior open bite. When the overbite is incomplete, imagine the long axis of the lower incisors and extend until it reaches the upper incisors. This shows where the lower

Examination and diagnosis  109 incisors would occlude if the overbite was complete and enables incisor classification. This is a class II division  incisor relationship. Buccal segment relationship is described using the Angle’s classification (I, II, or III) and is defined further as mild, moderate, or severe by describing the cusp/embrasure discrepancy as a fraction (¼, ½, ¾, ) of a premolar width. C This patient’s main problem is the vertical dimension. The lower anterior facial height is increased, associated with an anterior open bite. It is extremely important to establish control of the vertical dimension early during treatment and to ensure that the molars are not extruded. If this occurs, it will make the anterior open bite and class II skeletal relationship worse. Therefore, treatment mechanics should try to intrude the upper buccal segment teeth. This can be attempted by using temporary orthodontic implants sited in the buccal and palatal alveolus and running traction to them from the buccal segment teeth or high pull headgear to intrude the upper molars. Another option is to use a transpalatal arch that stands slightly proud of the hard palate, in the hope that tongue pressure will effect some molar intrusion. Class II traction should be avoided, as this will extrude the lower molars. D The presenting malocclusion has features indicative of problems with long-term stability post treatment. The anterior open bite may relapse, as orthodontics will have only a minimal effect on the aetiological factor, the increased vertical dimension. It has been found that the greater the skeletal contribution to the open bite, the worse the prognosis for stability. A mild open bite at the start of treatment has a greater chance that the result will be stable. Overjet reduction is likely to be stable if the lips are competent at the end of treatment, with the lower lip covering the incisal third of the upper incisors at rest. E If the skeletal pattern is of such severity that orthodontic treatment will not be successful, or the aesthetic requirements cannot be satisfied with orthodontic treatment alone, then a bimaxillary osteotomy can be offered. This will involve a differential impaction of the maxilla, so that the posterior teeth are impacted to a greater extent than the anterior teeth, resulting in a tipping of the occlusal plane and closure of the anterior open bite. The mandible would be advanced to correct the mandibular retrognathia.

Answer 2.6 A Anteroposteriorly, this patient has mandibular retrognathia and demonstrates a severe class II skeletal relationship. She has a good nasal profile and her nasolabial angle is within normal limits, at about 90°. This suggests that the maxilla is correctly positioned in the anteroposterior dimension. The Frankfort–mandibular planes angle is increased due to reduced posterior face height and increased anterior face height, as a consequence of a backward rotating facial growth pattern. At rest, she has incompetent lips. However, when bringing her lips together, the mentalis muscle appears tense, suggesting that she can only achieve lip competence with effort, i.e. lip strain. Transversely, her face is symmetrical. B In occlusion, there is a slightly increased overjet and a mild anterior open bite. Her upper incisors appear to be of average inclination but her lower incisors are proclined and her buccal segment relationship is essentially class  bilaterally. This is surprising, given the underlying class  II skeletal base relationship and suggests (correctly) that she has had orthodontic treatment previously. The lower anterior facial height is greatly increased and the anterior open bite has occurred simply because incisors have run out of eruptive potential. C As a 20-year-old female, her growth will have ceased, therefore the mandibular retrognathia cannot be addressed by orthodontic means alone. Conventional orthodontic treatment cannot reduce her vertical dimension or encourage her mandible to autorotate to mask her class  II skeletal relationship (although some clinicians advocate the use of temporary anchorage devices as a means of establishing upper molar intrusion

110 

Illustrated Questions in Orthodontics

and correcting the anterior open bite). In order to reduce the overjet with routine orthodontic treatment, space would be needed from extractions in the upper arch. This would be detrimental to her facial profile, since the extraoral examination determines the anteroposterior position of the maxilla to be correct, and would not address the lack of overbite. Hence, orthodontic ‘camouflage’ of her class  II skeletal relationship is not appropriate. In this case, optimum management is with a combined orthodontic/ orthognathic approach. This would involve a bimaxillary procedure to impact the maxilla, thereby reducing the vertical height and allowing lip competence to be achieved more easily. Simultaneously, the mandible would be advanced to produce a class I incisor relationship. In addition, an advancement genioplasty may also be needed to create a more normal profile. Even if she was sufficiently young at presentation to have mandibular growth potential, the severity of her skeletal base is such that an orthodontic approach may not be successful and she/her parent should be warned of this prior to starting treatment.

Answer 2.7 A This young boy is in the early mixed dentition and has a moderate class II division  incisor relationship. He has a mild class II skeletal relationship with an average vertical dimension. B He has incompetent lips, and the lower lip is trapped behind the upper incisors. He has an acute nasolabial angle (normal range is 90–0°) and a pleasing nasal profile. The soft tissue chin point (soft tissue pogonion) is normally related to the forehead (soft tissue nasion). Soft tissue pogonion should lie within 4 mm of a vertical plane dropped from soft tissue nasion. In this instance, his chin point (soft tissue pogonion) disguises the underlying skeletal discrepancy somewhat. This is quite a common occurrence. C The principal cause of the increased overjet is the soft tissue behaviour. The upper incisors have been significantly proclined by the lower lip that is positioned palatally to the upper labial segment at rest. This is called ‘lip trapping’. The aims of treatment here are to reduce the overbite, reverse the lower lip trap, and reduce the overjet. Increased mandibular growth relative to maxillary growth will compensate for the mild skeletal discrepancy. Successful reduction of the overjet will depend on alteration of the soft tissue behaviour. On completion of treatment, the lower lip must lie anterior to the upper incisors, whose corrected position will then be retained by lower lip control. This is achieved when the lips are competent (held together) at rest without conscious effort, i.e. without lip strain. D Lip competence is important for several reasons. Firstly, the lower lip acts as a retainer, maintaining the position of the upper incisors and minimizing the overjet. Secondly, during swallowing, lip competence creates an anterior oral seal, which keeps the bolus within the oral cavity. Finally, lip competence minimizes mouth breathing.

Answer 2.8 A Anteroposteriorly, this patient has a class II skeletal relationship, with the mandible set back relative to the maxilla (soft tissue B point is posterior to soft tissue A point) and her Frankfort–mandibular planes angle is reduced. B The angle ANB is equal to or greater than 5° in a class II skeletal relationship. C The class II division 2 type malocclusion usually has a class II skeletal relationship, but this can be skeletal I or III. The cranial base angle (formed by the lines joining nasion, sella, and basion) is often obtuse, which causes mandibular retrognathia, and the cranial base is long, resulting in maxillary prognathism. The maxilla is short, broad, and forwards relative to the mandible, which may result in a scissorsbite. The lower face height is usually reduced,

Examination and diagnosis  111 with reduced gonial angle. This is the angle formed by the ascending (ramus) and horizontal (mandibular plane) parts of the mandible (e.g. a square face). Likewise, the Frankfort– mandibular planes angle is usually reduced. A high resting lower lip line is common in this type of malocclusion, due to the reduced lower anterior face height. There is a marked labiomental fold and the masseter muscles may produce an increased bite force. D This patient has a strong chin, which masks her mandibular retrognathia.

Answer 2.9 A This photograph demonstrates a ¾ unit class II buccal segment and canine relationship. The upper incisors are very retroclined, with minimal overjet. The overbite is increased and complete to the labial gingivae, with signs of trauma. B This is a class II division 2 incisor relationship. This is defined by the incisor relationship where the lower incisor edges occlude palatally to the cingulum plateau of the upper incisors and the upper incisors are retroclined, with a minimal overjet, although this may be increased. The retroclination of the upper incisors is the key to distinguishing between a class II division 2 and class II division  incisor relationship. C The key dental features of a class  II division 2 malocclusion are retroclined upper and (usually) lower incisors, which result in an increased interincisal angle and deep overbite. The overjet is usually reduced and the buccal segments are often class II. A scissorsbite is common in the first premolar region, due to the broad maxilla. The crown–root angle of the upper incisors may be reduced and they may have a poorly defined cingulum. D The interincisal angle and lower incisor position, if retroclined, must be corrected after treatment to give the best chance of stability. In order to reduce the overbite with maximum stability, the lower incisor edges should lie anterior to the upper incisor root centroid after treatment. Treatment of the class II division 2 malocclusion is one of the few instances where it is acceptable to procline the lower labial segment. This is because the retroclined upper incisors have caused the lower incisors to retrocline during growth and treatment mechanics aim to procline the lower incisors to a normal inclination. E The high resting lower lip line is the primary aetiological factor in the retroclination of the upper incisors. The upper lateral incisors are often trapped on top of the lower lip, becoming mesially angulated and mesiolabially rotated.

Answer 2.0 A This patient has a moderate class III skeletal relationship with an average Frankfort–mandibular planes angle. There is an obtuse nasolabial angle, which suggests that the maxilla is retrognathic. However, the skeletal relationship assessment should be made in conjunction with an assessment of the incisor inclination. If the lower incisors are found to be retroclined, or the upper incisors proclined, then the skeletal relationship will be more class III than appears clinically. This is because the incisor position has compensated for the underlying skeletal base discrepancy, to try to create a functional incisor relationship, i.e. so the person can incise. The net result is that this disguises the severity of the skeletal pattern. B This case is a mild class III incisor relationship with mild crowding. The buccal segment and canine relationship is a ½ unit class III. The overjet and overbite are reduced, with the upper right lateral incisor in crossbite with the lower right canine. The lower incisors appear to be retroclined, compensating for an underlying class III skeletal relationship. C Patients with a class III incisor relationship usually have a class III skeletal relationship. The cranial base angle may be acute, leading to a forward position of the mandible, or the

112 

Illustrated Questions in Orthodontics

cranial base may be short, leading to maxillary retrognathia. The mandible may be large, the maxilla small, or a combination of both. The maxilla tends to be small and narrow relative to the mandible, which tends to be broad. Hence, crossbites are common. The gonial angle is often obtuse and the Frankfort–mandibular planes angle and lower anterior facial height are commonly increased. The pattern of mandibular growth is usually in a backward and downward direction. D The aims of orthodontic treatment of this class III malocclusion are to: i relieve the crowding ii align the teeth iii correct the buccal segment relationship to class I iv if possible, to increase the overjet and overbite. The upper incisors will need to be proclined and the lower incisors retroclined beyond average cephalometric values. E Unfavourable mandibular growth, especially vertically, will limit the correction of the overjet and overbite.

Answer 2. A These photographs demonstrate forwards displacement of the mandible on closure to achieve a comfortable occlusion. The patient can reach edge to edge incisor contact, but then displaces forwards into centric occlusion. It is important to diagnose displacements on closure as this can disguise the true skeletal relationship. For example, this patient assessed in the displaced position will appear to have a more severe class III skeletal relationship than is correct. The true skeletal relationship will appear more amenable to correction orthodontically than the displaced position. A lateral mandibular displacement will suggest a facial asymmetry that is absent when the patient is assessed in the undisplaced position. B This patient has missing upper lateral incisors, therefore the upper buccal segments are one unit forwards relative to the mandibular teeth. If the lateral incisors were present and aligned, the buccal segment relationship would be considerably more class III. Both the upper and lower incisors appear to be retroclined. The lower incisors are compensating for the underlying class III skeletal relationship, whilst the upper incisors have been trapped in crossbite by the lower incisors and, therefore, are not at their normal inclination. If the upper lateral incisors were present, this may have prevented the upper central incisors from becoming trapped by the lower incisors, as the upper central incisors would probably have been in a more normal position.

Answer 2.2 A This patient has a mild class III skeletal relationship anteroposteriorly and increased lower anterior facial height. As a result, the mandibular plane is steep and the Frankfort–mandibular planes angle is increased. This is an abnormal vertical skeletal relationship. B The class III skeletal relationship has an ANB angle that is equal to or less than º. C In this type of skeletal relationship, the maxilla is positioned posteriorly relative to the mandible. This may be due to a small maxilla (maxillary retrognathia), or a large mandible (mandibular prognathism), or a combination of both. Alternatively, the mandible may be positioned anteriorly due to an acute cranial base angle. D This patient has had excessive anterior facial growth relative to the posterior facial growth. As a result, the mandible has rotated backwards (posterior growth rotation) carrying the mandibular incisors vertically away from the upper incisors. Growth of the

Examination and diagnosis  113 alveolar processes has been unable to keep pace with mandibular growth and the teeth in the labial segments have achieved their full eruptive potential. This is not sufficient to achieve an overbite. The combination of these factors has resulted in the vertical gap between the incisors. This is called an anterior open bite. Other causes of anterior open bites include habits such as thumb sucking, or soft tissue factors such as tongue size or position. However, neither of these factors was involved in the aetiology of the anterior open bite in this patient. E Given the severity of the anterior open bite and that the main aetiological factor is the skeletal relationship, a combination of orthodontics and orthognathic surgery is required for correction of the malocclusion.

Answer 2.3 A This patient has a class III skeletal relationship with increased lower anterior facial height and an increased Frankfort–mandibular planes angle. B In this case, the buccal segment relationship demonstrates a severe class III relationship, which is a full unit class III. The upper first molar only just occludes with the distal cusp of the lower first molar. The upper premolars are in crossbite with the lower first molar. In this example, the lower incisors occlude anterior to the cingulum of the upper incisors (in fact, in anterior crossbite). Therefore, the incisor relationship is class III, demonstrating a reverse overjet and reduced overbite. The lower incisors appear to be retroclined, and both upper and lower incisors are slightly imbricated. C The incisor relationship demonstrates the phenomenon of dentoalveolar compensation for a class III skeletal relationship. In an attempt to achieve a normal incisal relationship, the upper incisors have proclined. In this case, the upper incisors (ui) were at 20° to the maxillary plane. The lower incisors (li) were retroclined to the mandibular plane at 72°. The compensated incisor positions have resulted in a more normal soft tissue profile than if the incisors were positioned at an average inclination to the dental bases (normal Caucasian ui/MPA = 09° +/– 6°; normal Caucasian li/ MnPA = 93° +/– 6°). If the incisors were normally inclined, the resulting profile would show greater mandibular prognathism. This example demonstrates how tooth position can camouflage the severity of the underlying skeletal relationship.

Answer 2.4 A This patient has a marked mandibular asymmetry with the chin point off to the left. B Facial asymmetries can often worsen with continuing growth and what might have been a mild problem at a young age can cause significant distress when the patient is older. Orthognathic surgery may be the only answer to correct the aesthetics and it may be best to delay orthodontic treatment until it is clear that growth has slowed and the asymmetry is not worsening. C Mandibular displacements on closure can worsen the apparent skeletal discrepancy. As lateral displacements can give the false impression that a transverse asymmetry exists, these must be identified in order to diagnose the malocclusion correctly. D A lateral mandibular displacement in a child in the mixed dentition often results because of a transverse discrepancy between the maxillary and the mandibular dentition. Commonly, this is the result of a digit sucking habit, causing upper arch narrowing, leading to a premature contact on closure. Palatal lateral incisors and deciduous canines are also common causes of occlusal interferences and, thus, premature contacts. Treatment frequently involves a combination of upper arch expansion with a removable appliance and elimination of the premature contact, or extraction of the deciduous tooth. Digit-sucking habits should be discouraged.

114 

Illustrated Questions in Orthodontics

Answer 2.5 A Both these cases demonstrate bimaxillary proclination where the upper and lower labial segment teeth are proclined. The incisor relationship is class I. B This problem is more common in Afro-Caribbean and oriental ethnic groups. C Tooth position is largely determined by the forces of the orofacial musculature at rest, i.e. the resting position of the lips and tongue. Lips that are full and everted tend to exert less pressure on the teeth than lips that are thin and strap-like. Therefore, pressure from the tongue is not resisted by lip pressure and the teeth procline. D Patients with bimaxillary proclination, who ask for treatment, often dislike any associated spacing and sometimes complain that their ‘teeth stick out’. Occasionally, the overjet is increased, but often it is normal and, in these cases, the patient needs to be reassured that they are mistaking the inclination of the teeth for an increased overjet. Some patients dislike their full profile and others wish to achieve lip competence. E The stability of treated bimaxillary proclination is variable, as retroclination of the labial segments will infringe on the tongue space. Treatment should be undertaken with caution and the patient warned, in advance, that long-term stability might only be possible with long- term retention. Patients who do best are those presenting with crowding and a need for premolar extractions. This relieves crowding and enables retroclination of the incisors. Premolar extractions should be avoided in spaced or aligned cases, as extraction spaces will often re-open post treatment under the influence of the tongue. Patients who are able to achieve lip competence after orthodontic treatment also have a greater chance of stability than those who had competent lips at the start of treatment. Often prolonged, indefinite retention is required.

Answer 2.6 A (i) 0 years (ii) .5 +/–± 2 years B The deciduous canines are retained in the arch and all other permanent teeth, except third molars, have erupted. In this case, the permanent canines were impacted palatally. C Palpate the canine region to assess the canine position. If the canine position cannot be determined, take parallax radiographs (DPT and upper anterior occlusal view or two periapical films from different angles) to determine the canine position radiographically. If the canine position alters in the same direction as the X-ray tube shift, then it lies palatally. If the canine moves in the opposite direction to the tube shift, then it lies buccally. If there is no change, then it is in the line of the arch. This method of identifying tooth position can be remembered as the ‘SLOB’ (same lingual, opposite buccal) rule. In complex cases, the use of cone beam tomography may provide additional information on the tooth position and root resorption of adjacent teeth. D There are many factors to take into account when treatment planning for palatally impacted canines. These include: i patient age and attitude towards treatment ii state of general and dental health iii presenting malocclusion iv position of the canine and the severity of the impaction v root resorption of teeth adjacent to the canine vi extent of crowding vii lateral incisor/premolar contact.

Examination and diagnosis  115 General feedback In general terms, the more palatally positioned the canine, the less chance there is of successful orthodontic alignment. For example, impacted canines that lie horizontally, with the crown near the midline, will be more difficult to align than those of vertical orientation. The position of the canine root apex must be favourable. If the canines are normally positioned, then no treatment is necessary, although extraction of the deciduous canines should be considered to hasten eruption of the permanent canines. If the canines are impacted, the following should be considered: Interceptive management For a child aged 0–3 years with uncrowded arches, extract the deciduous canines and maintain the space, if there is sufficient space, or create space in crowded cases, for normalization of the upper canines. This is successful in 65–88% of cases presenting with palatally impacted upper permanent canines. Active management to align the canines The impacted canines can be exposed surgically (open exposure) and aligned using an orthodontic appliance. Some clinicians prefer a gold chain to be bonded to the canines at the time of surgery and the mucosal flap to be replaced (closed exposure). If the canines are buccally positioned and high, apically repositioned flaps for exposure are not feasible. These malpositioned canines require a flap to be raised and a bond with a gold chain to be attached to the canine. A fixed appliance is then used to apply traction to the chain, which brings the tooth down into position. Surgical removal of the impacted canines Removal is a realistic option if the canines are very unfavourably positioned and lateral incisor/premolar contact exists or can be created orthodontically. Passive management Impacted canines can be kept under radiographic observation. This is not the ideal solution as the deciduous canines will eventually be lost and prosthetic replacement will need to be considered.

Answer 2.7 A This is a postoperative photograph of palatally impacted permanent canines that have been surgically exposed. This allows access to bond an attachment, such as a bracket or eyelet, which can then be used to apply traction to the impacted teeth. Additionally, this patient has molar separators in place prior to placing molar bands. An alternative method of managing impacted canines is by direct bonding with eyelet and gold chain at the time of surgery and flap replacement. Surgically exposing the crown, rather than the gold chain and flap replacement approach, has three principal advantages. First, as the tooth is bonded postoperatively, a dry field should be easier to achieve and so the bond may be more successful. Second, if the canine is allowed to erupt post exposure, it may be possible to bond the labial surface rather than the palatal surface, which limits the canine rotating whilst being aligned. Finally, if the bond fails, it is rapidly noticed and repaired. However, in the hands of an inexperienced surgeon, the exposure may be incomplete and the canines may be quickly covered by granulation tissue. If the canine is deeply impacted, an open exposure may not be possible and a replaced flap would be preferable. Either method can work very successfully, but both techniques require careful surgical technique. B Impacted maxillary canines occur in approximately 3% of the population.

116 

Illustrated Questions in Orthodontics

C This problem is more common in females, with a ratio of females: males of 85:5. D Several factors are associated with impacted maxillary canines. It is a familial condition, predominating in females. There are associations with class  II division 2 malocclusions, deep bites, and small or missing upper lateral incisors. E Root resorption of the upper lateral incisors will occur in 2% of cases with impacted maxillary canines, especially in girls under 4 years of age. In severe cases, the roots of the upper central incisors can also be resorbed.

Answer 2.8 A This is a lateral open bite, also known as a posterior open bite. B This rare condition has a number of possible causes, which may be of skeletal, soft tissue, or dental origin. It may be a combination of all three. Often, the skeletal vertical dimension will be increased and development of the alveolar process will not have kept pace. Rarely, unilateral condylar hyperplasia may be the cause. Tongue position (as in this case) may prevent full tooth eruption. This may be due to the tongue being large or premature loss of the first molars allowing the tongue to spread into the extraction space. Finally, it may arise from primary failure of tooth eruption or arrested tooth eruption. C It is important to be sure that the affected teeth are not ankylosed, as these cannot be extruded orthodontically. The net effect could be intrusion of the adjacent, unaffected teeth. Ankylosed teeth have a distinctive sound when percussed that is duller than adjacent teeth. It is important to check this before and during treatment. Additionally, careful measurement of the lateral open bite should be made at each visit, as a progress check, and adjacent tooth position should be monitored to ensure that the overbite is maintained. If tooth contact is achieved orthodontically, this result is likely to be unstable and the patient should be forewarned of this at the start of treatment.

Answer 2.9 A This is a child in the early mixed dentition who has an asymmetrical anterior open bite and unilateral crossbite tendency on the right. The fraenal attachment is low and fibrous and may be contributing to the diastema. B The asymmetrical nature of the malocclusion suggests that the aetiology is due to a persistent habit, in this instance thumb sucking. C Digit sucking may have several effects on the dentition and skeletal relationship, depending on the duration and severity of the habit. Habits ceasing prior to the eruption of the permanent dentition will not have any long-lasting effects, although prolonged dummy use or digit habits can impact on the deciduous dentition, analogous to dental effects seen on the permanent dentition. Generally, the overjet is increased and asymmetrical, whilst the overbite is reduced and incomplete. The upper incisors are proclined and the lower incisors are retroclined, usually asymmetrically. The tongue is displaced by the thumb and so held at a lower position than normal. Without tongue pressure, the maxilla narrows and a unilateral crossbite tendency is common. A  lateral mandibular displacement on closure may be present. The mandible is held open, resulting in excessive eruption of the posterior teeth, a backward growth rotation, and a more class II skeletal relationship. D This child initially needs to be encouraged to stop thumb sucking. Sometimes, an explanation may be sufficient, which can be combined with a reward chart for younger children. A  product called ‘Thumbguard’ (available from www.​thumbguard.co.uk) prevents the creation of an orodigital seal and helps to stop an established habit. When the permanent dentition is more established, an upper removable appliance can be fitted to expand the upper arch, and this may stop the habit. Fixed habit breakers, such as the hayrake

Examination and diagnosis  117 appliance or ‘Bluegrass’ appliance (http://www.haskellbraces.com/thumb-sucking), may be considered. Finally, physical deterrents such as white cotton gloves, Sticking plasters, or ‘Stop ‘n’ Grow’ lotion (available from chemists), may help to stop the habit. However, committed thumbsuckers will carry on, even into adulthood.

Answer 2.20 A This is a child in the early mixed dentition with the upper central incisors trapped in crossbite with the lower central incisors which are labially displaced and demonstrate gingival recession. This is called an anterior crossbite. B This is a significant problem as the periodontal attachment of the lower incisors can be damaged. C The upper incisors need to be proclined with an upper removable appliance. The lower incisors may need to be actively retroclined. D Various designs are possible for this problem. This is the design used:

Upper removable appliance i cantilever springs behind the upper incisors in 0.7 mm stainless steel wire ii Adams cribs on the upper first permanent molars in 0.7 mm stainless steel wire and Jacksons cribs on the deciduous first molars in 0.6 mm stainless steel wire iii posterior bite capping iv baseplate.

Answer 2.2 A The right maxillary canine and first premolar have been transposed. B Transpositions are not a common problem. However, they almost always involve the canine. Transposition of the canine and first premolar is the most frequently occurring (7% of recorded cases). It is more common in females than males and is more frequent on the left. C The root position of the affected teeth has a major influence on whether or not to accept the transposition. Sometimes only the crowns have changed position and this is described as a pseudotransposition, which is easier to correct. If the roots have transposed, then their position is best accepted. Other factors that influence treatment planning are the degree of crowding, type of malocclusion, and patient motivation. D The aetiological factors contributing to this problem include the early loss of deciduous teeth and genetic predisposition.

Answer 2.22 A The upper right central incisor is missing and space has been lost. Investigations should determine if it is impacted or has been avulsed, as in this case. B Space maintenance before orthodontic treatment can be difficult, especially if the incisor is lost at a young age. An upper removable appliance (URA) with an artificial tooth should be provided. It is optimistic to expect acrylic alone to preserve the space entirely. Although wires detract from aesthetics, C clasps, mesial and distal to the missing tooth, will give the best chance of total space maintenance. C Aesthetics can be maximized during orthodontic treatment by maintaining the URA as a spoon denture for as long as possible. Alternatively, an acrylic tooth may be bonded with a bracket and ligated to the archwire. This is difficult with the initial light wires since the

118 

Illustrated Questions in Orthodontics

acrylic tooth tends to spin on the wire. If the incisor is lost during active treatment, then it can be decoronated and a bracket bonded to the enamel. The natural crown may then be ligated to the wire. D Definitive restorative options should be discussed with the dentist who will provide the restorative treatment before orthodontic treatment begins. The dilemma at the start of treatment is whether to open or close the space. It is possible to bring the lateral incisor into the central incisor position and then crown it to mimic the central incisor. The aesthetics of this are rarely satisfactory as the gingival margin and crown contour look abnormal, although adhesive composite restoration can mask this to a degree. The best aesthetics will be achieved by opening the space and providing an artificial replacement. Options include:

Denture This is satisfactory in the short term as the dentition becomes fully established, but it is not ideal for a single tooth replacement.

Bridgework Conventional bridgework involves crowning adjacent teeth to support the pontic. This is very destructive and is only indicated if the adjacent teeth are heavily restored. Adhesive bridgework is preferred, providing the overbite allows for the placement of metal wings. In general, a cantilever design has the greatest success rate.

Implant Single tooth implants are increasingly more popular. However, they are expensive and cannot be used until growth has ceased since implants do not erupt with continued alveolar development. The final restoration must be considered carefully at the start of treatment since requirements are different for each option. For example, the roots adjacent to the space need to be parallel or divergent for implant placement but not for an adhesive bridge.

Answer 2.23 A This patient, in the early mixed dentition, has infraoccluded deciduous molars and an anterior open bite. The first molars are the only teeth in occlusion. B The deciduous molars have become ankylosed. Teeth become ankylosed when the process of resorption and repair becomes imbalanced. As deciduous teeth are shed, their roots undergo resorption and repair simultaneously. If the rate of repair exceeds resorption, then the roots can fuse to surrounding bone. As the surrounding teeth and alveolar processes continue to develop normally, the ankylosed teeth are left behind and become infraoccluded. Commonly, single teeth (usually second deciduous molars) are affected and this is often due to the lack of a permanent successor. However, in this case, where several teeth in each arch are affected, there appears to be a generalized disturbance in eruption and the affected teeth may have never reached the level of the normal occlusal plane. This is referred to as a primary failure of eruption. C If the deciduous molars have an eruption disturbance, then the prognosis for the permanent dentition is poor, as it is likely that it will be similarly affected. It is not possible to extrude affected teeth orthodontically; therefore, a restorative approach, to build up the teeth to the level of the occlusal plane or to provide an overdenture, may need to be considered. D The anterior open bite may be due to: i skeletal factors, e.g. increased lower anterior face height ii soft tissue factors, e.g. forwards tongue posture or atypical swallowing pattern

Examination and diagnosis  119

iii habits, e.g. digit sucking iv combination of the above.

Answer 2.24 A The upper right central incisor is missing and space loss has occurred. There is contact between the upper left central and upper right lateral incisors. The upper left central incisor has drifted to the right, across the dental midline (reference its position to the lower dental centreline), and has displaced the labial fraenum. B This problem should be investigated by:

Taking a full history Has the tooth been previously extracted or avulsed? Is there a history of trauma to the deciduous predecessor?

Dental and orthodontic examination Is the missing incisor palpable? What is the standard of dental health? What are the other presenting features of the malocclusion?

Radiographs Radiographs identify the presence and position of the impacted tooth and also identify any associated pathology. In order to use the parallax method of identification to identify the position of the missing tooth, radiographs should be taken in two planes. A dental panoramic tomogram and upper anterior occlusal view would be ideal. Alternatively, if clinical interest is focused only on the upper incisor region, then two periapical radiographs, taken at different angles, would suffice. In complex cases, the additional information from cone beam tomography views might be helpful. C Firstly, a diagnosis must be established as the upper right central incisor could be missing or impacted. If it has been lost, then a decision needs to be made as to whether to keep the existing malocclusion and disguise it by restorative means or whether to open space for replacement of the central incisor. Full discussion with a restorative dentist, the patient, and parents, if necessary, must be undertaken before a decision is reached. In fact, in this instance, the tooth was impacted. The management of this problem is dependent on many variables which need to be considered collectively. These include patient motivation, degree of tooth impaction, presenting dental health and malocclusion, quality of the impacted and adjacent teeth (e.g. is the root dilacerated or resorbed?), and the standard of available surgical and restorative support. In general, the treatment options to be considered for the impacted tooth are:

Alignment Expose surgically and bond the impacted incisor, or use a gold chain and replacement flap procedure before aligning the impacted tooth with fixed appliances.

Removal The impacted tooth could be removed and the lateral incisor built up to mimic the central incisor. This is not the best aesthetic option, but it does avoid the need for orthodontic treatment. Alternatively, space could be created orthodontically for the restorative replacement of the central incisor.

Transplantation This is dependent on the impacted tooth being removed intact.

120 

Illustrated Questions in Orthodontics

No active intervention Keep the impacted tooth under periodic radiographic review. Space to accommodate the impacted incisor will need to be created orthodontically, beforehand, if it is to be aligned or transplanted. D Aetiological factors which may have caused this problem include: i trauma, e.g. in the deciduous dentition, trauma can result in damage to the permanent tooth germ, which may develop a dilacerated root and fail to erupt ii midline supernumerary iii pathology, e.g. dentigerous cyst.

Answer 2.25 A This radiograph demonstrates hypodontia. B This patient is missing all second premolars, the lower left first premolar, and all third molars. The image of the upper right first premolar and upper right canine are superimposed. Theoretically, the third molars could still start calcifying up to the age of 0 years, but given the degree of hypodontia evident here, it is highly unlikely they will develop for this patient. C The most commonly missing teeth in the Caucasian population are third molars (25%), second premolars (3%), and upper lateral incisors (2%). D Lower central incisors are the most commonly missing tooth in the Japanese population. E Cleft lip and palate, Down’s syndrome, hypohidrotic ectodermal dysplasia, and orofacial digital syndrome are all associated with missing teeth. However, most patients with hypodontia do not have additional abnormalities. In these hypodontia cases, paradoxically, there may also be extra teeth and the other adult teeth are often smaller.

Answer 2.26 A This photograph demonstrates missing upper lateral incisors. The maxillary canines are in contact with the upper central incisors. B The incidence of developmentally missing upper lateral incisors is approximately 2%. C The main decision to be made is whether to open space and provide prosthetic replacements for the lateral incisors or to keep the canines alongside the central incisors, as shown here. This decision can only be made after a number of factors have been taken into consideration. These include: i the presenting malocclusion ii colour and shape of the canines iii the available restorative back-up and who will bear the long- term cost iv patient motivation. D The maxillary canines can be aesthetically modified by: i grinding, e.g. the cusp tip can be ground to create an incisal edge ii restorative build-up, e.g. composite added to the labial aspect or incisal edges of the canines can improve their appearance iii position, e.g. the tip and torque expressed by a pre-angulated, pre-torqued bracket can be manipulated by altering the bracket position so as to move the canine so that it resembles a lateral incisor.

Examination and diagnosis  121

Answer 2.27 A This overbite is increased and complete to the palate. The inflamed palatal mucosa has been stripped from the left upper central and lateral incisors, resulting in exposure of the root. This is called a traumatic overbite. Additionally, the upper left central incisor is discoloured and has a palatal restoration, suggesting that it has been root-treated. B The deep overbite is caused by skeletal disproportion, commonly mandibular retrognathia, with the lower incisors set back further than ideal. As a result, they occlude with the hard palate or the gingival margin instead of the cingulum plateau of the upper incisors. The accumulation of plaque and the deep traumatic overbite are a lethal combination for periodontal support and bone loss. C A regime of meticulous oral hygiene should be established and the problem kept under review. Reduction of the overbite should be considered and, in a growing child, is usually straightforward, but may only be achievable through orthognathic surgery for an adult.

Answer 2.28 A The overjet is greatly increased in the second photograph compared to the first. This discrepancy is due to the patient posturing her mandible forwards in the first image to disguise her underlying skeletal discrepancy. B During examination, it is extremely important to ensure that the patient’s mandibular condyles are correctly seated in the articular fossae in order to diagnose the true malocclusion and skeletal base relationship. In this instance, what may have been regarded as an acceptable overjet in the first photograph is revealed to be much more severe in the rest position and required an orthognathic correction.

chapter three Treatment planning Answer 3. A B

General aims of orthodontic treatment are to: i relieve crowding ii level, align, and co-ordinate the dental arches iii correct the canine relationship to class I iv correct the overbite and overjet v correct the buccal segment relationship to maximum intercuspation vi close space vii retain correction. In general, orthodontic treatment aims to establish a class I incisal relationship. With the teeth at normal inclination, this can be achieved only if the underlying skeletal relationship is, or is very nearly, class I in the anteroposterior plane. If the skeletal relationship is not class I, then it needs to be corrected, or the tooth inclination needs to be manipulated to disguise the underlying skeletal discrepancy. This is called orthodontic camouflage and is at its most successful if the skeletal discrepancy is mild. Therefore, when a patient is assessed, the clinician must decide if the existing skeletal relationship will allow a normal incisor relationship to be achieved or if the skeletal relationship must be modified. This can be attempted through choice of treatment mechanics, such as a functional appliance, the application of headgear, or through orthognathic surgery at the end of growth. It must be stressed that the alteration of the skeletal relationship through non-surgical means is achievable only to a limited extent. Often, correction of a class II skeletal relationship has occurred simply because a patient has grown favourably and not because the orthodontist has been able to influence growth.

General feedback Vertically, the skeletal relationship influences the degree of overbite found. Therefore, a patient with reduced vertical dimensions tends to have an increased overbite and treatment planning must include how this overbite will be reduced. Conversely, a patient with increased vertical dimension and reduced overbite will need careful treatment planning so that the overbite is not further reduced by the treatment mechanics. Finally, to achieve good facial aesthetics and maximal buccal segment interdigitation, the upper and lower dental centrelines must be coincident with each other and the facial midline. Therefore, the transverse skeletal plane must be assessed for underlying asymmetry and also for cants in the occlusal planes which would indicate asymmetrical growth. If an asymmetry exists, treatment is best delayed until growth has ceased and then a decision can be made as to whether it is to be accepted, with compromised facial and occlusal appearance, or treated by surgical means. C Multiple factors influence the decision as to whether or not dental extractions are required to treat a malocclusion. However, in patients with a reduced vertical dimension, mandibular space closure is more difficult than in those with an increased vertical

Treatment planning  123 dimension. Therefore, in general, mandibular extractions should be avoided, where possible, for patients with a reduced vertical dimension.

Answer 3.2 A Growth potential is significant when assessing unfavourable features of the presenting skeletal relationship. For example, mandibular growth will help the correction of a class II skeletal relationship but will hinder camouflage of a class III skeletal relationship. Growth will help reduce a deep overbite but may worsen a reduced overbite. Likewise, an asymmetry is likely to worsen if further growth occurs. Therefore, potential growth must be considered when planning treatment. For example, treatment of a class II malocclusion will be aided by growth, whilst treatment of a high angle class III case and those cases with asymmetry are best delayed until growth has slowed and the clinician can truly assess the final malocclusion. B Treatment aims most helped by favourable growth are the reduction of overbite and overjet, space closure, and correction of a retrognathic profile. C There is considerable individual variation in the timing of the pubertal growth spurt. It is nearly two years earlier in girls than boys. Girls tend to have a growth spurt between the ages of 0.5–2.5 years and boys, between 2.5–4.5 years. D Growth studies have established that both jaws rotate during growth. A backward mandibular growth rotation is described when the mandibular border and chin point rotate downward and backward. i This will worsen a class II skeletal relationship, as B point on the mandible is carried further behind A point on the maxilla. ii Additionally, a backwards mandibular rotation will reduce the overbite. iii Treatment mechanics should be aimed at minimizing unwanted effects. Mandibular molar extrusion will worsen a backward rotation, therefore class II traction, which extrudes lower molars, should be avoided if possible and maxillary molars can be actively intruded to encourage rotation of the mandible in a forward direction.

Answer 3.3 A A normal overbite is important for satisfactory masticatory function, i.e. ensuring separation of the posterior teeth during mandibular protrusion and facilitating incision. Overbite is also important for stability following the correction of anterior crossbites. Ideally, the upper incisors should overlap the incisal third of the lower incisors and the lower incisal edges should occlude with the palatal cingulum of the upper incisors. B The vertical skeletal dimension and degree of overbite are inversely related. As the anterior face height increases, the overbite decreases. Eventually, the alveolar processes reach the limits of growth and tooth eruption fails to compensate for the increased vertical dimension and an open bite may result. Conversely, people with reduced anterior face height are likely to have a deep overbite, as the alveolar processes are closer together. C The overbite must be reduced before the overjet can be reduced in order to stop anterior tooth contact and create space for the upper incisors to move into, assuming the facial profile allows them to be retracted. D Overbite reduction can occur in two ways, i.e. by extrusion of buccal segment teeth or intrusion of the incisors. Often, it is achieved by a combination of both. Tooth extrusion requires bony support and alveolar basal bone growth occurs with greater volume and speed in a growing child than in adult patients. E Overbite reduction may be achieved by:

124 

Illustrated Questions in Orthodontics

i the anterior bite plane of a removable appliance, which allows mandibular molar eruption ii molar eruption whilst the incisors are held in a class I relationship with a functional appliance iii correction of the interincisal angle in deep bite cases with retroclined incisors. This reduces the overbite automatically. iv extrusion of mandibular molars with class II traction applied to a fixed appliance v incorporating all lower molars into a fixed appliance vi applying curves of Spee to upper archwires and reverse curves of Spee to lower archwires vii J hook headgear to intrude the upper incisors (although this does increase root resorption) or cervical pull headgear to extrude the maxillary molars viii segmental intrusion mechanics with a fixed appliance ix surgical levelling.

Answer 3.4 A Crowding occurs when the space required to align the teeth is greater than the space available. This is called a space discrepancy. Mild crowding exists when the space discrepancy is 4 mm or less, a moderately crowded case has a space discrepancy of 5–8 mm, and a severely crowded case has a space discrepancy of 9 mm or more. Both upper canines are excluded buccally in this patient; therefore, 8 mm of space is required to align them. A little space will be available once the upper second deciduous molar is lost, but this will be insufficient to accommodate the canines. This case is severely crowded and will need upper arch extractions. B The following buccal segment relationships will result: i class I ii class III iii class II C Many factors must be taken into consideration when planning dental extractions. These can be classified as extraoral and intraoral factors. Extraoral factors are: i skeletal relationship, e.g. avoid lower arch extractions in cases with reduced vertical dimension ii soft tissue profile, especially the nasolabial angle. An obtuse nasolabial angle may be worsened by retraction of the upper incisors. Therefore, it is desirable to avoid upper premolar extractions, if possible, in these patients. iii growth potential, e.g. lower arch extractions should be avoided in skeletal III cases in whom growth is still active as this may compromise future orthognathic surgery. Intraoral factors are: i tooth number and quality, e.g. first premolars should not be removed without ensuring the second premolars are present, in a good position, and, ideally, of good quality ii degree and site of crowding iii magnitude of overbite and overjet iv dental centreline position relative to the facial midline v anchorage requirements vi the planned buccal segment relationship at the end of treatment. For instance, bilateral extractions in the lower arch only, in a patient with a full complement of permanent teeth, will create a class III buccal segment relationship, with unopposed terminal upper molars. Therefore, it is usually preferable to extract in the upper arch also, to keep the buccal segment relationship as class I.

Treatment planning  125

vii pathology, e.g. impacted canines may be too severely malpositioned to align orthodontically and may be extracted viii incisor inclination, e.g. extractions may be required for decompensation prior to orthognathic surgery.

Answer 3.5 A In general, treatment planning starts with the lower arch. In order to maximize treatment stability, the inclination of the lower labial segment should be the same at the beginning and end of treatment. Therefore, treatment planning should start with how to manage the lower arch malocclusion, while maintaining the lower incisors at their initial labiolingual inclination. Two principal exceptions to this general rule are class II division 2 cases and those with a digit habit. This is because the lower incisors have been artificially retroclined from their natural labiolingual position. Some clinicians start treatment planning by assessing the lower incisor position relative to the aesthetic line, A point to pogonion (APo), as it is thought that the best profile is achieved when the lower incisors rest on this line. However, this is not a guide to maximal stability. B The mandibular intercanine and intermolar widths are generally not altered during treatment since they usually relapse to their initial positions.

Answer 3.6 A This child is in the early mixed dentition. B The position of the upper permanent canines should be diagnosed by the age of 0 years. If they are not palpable in the buccal sulcus by this age, then radiographic examination should be performed. C Early unilateral loss of a deciduous canine is likely to cause a shift of the centreline to the affected side and this will prevent an ideal buccal segment interdigitation being achieved in the permanent dentition. An upper centreline shift can be very unaesthetic. The effects of early loss of deciduous teeth are most pronounced in crowded cases. D Early unilateral loss of a deciduous second molar is likely to cause the developing dentition to be crowded as the first permanent molar will drift or tip mesially. The succeeding second premolar may become impacted. E Extraction of the deciduous canine on the affected side and space creation, if the case is crowded, will help to normalize a maxillary permanent canine that appears to be palatally positioned.

Answer 3.7 A Unilateral extractions can worsen or cause malocclusion due to centreline shifts and the development of asymmetrical buccal segment occlusion. A balancing extraction is made to preserve symmetry and, therefore, limits adverse effects on the developing occlusion. Early unilateral loss of a deciduous canine or first deciduous molar has the greatest effect on the centreline and should be balanced by loss of a contralateral deciduous tooth. B A similar principle applies when teeth are lost in one arch only and an extraction is required in the opposing arch in order to limit deleterious effects on the occlusion. This is called a compensating extraction. For example, loss of a mandibular first molar prior to the eruption of the lower second molar may result in the maxillary first molar over-erupting. Therefore, consideration should be given to loss of the maxillary first molar at the same time. All the presenting features of the occlusion should be considered when planning extractions.

126 

Illustrated Questions in Orthodontics

C There are no rigid guidelines to follow when considering space maintenance and it should be planned for the benefit of the individual. However, in general, space should be maintained for avulsed upper central incisors and impacted canines following the loss of the deciduous canine. When deciduous molars are lost, a decision should be based on the child’s likely need for extractions of permanent teeth in the future. If they will not need extractions or will be severely crowded and need every millimetre of extraction space to treat the malocclusion, then space maintenance will be helpful to avoid further space loss. However, if extractions will be needed and the case is not severely crowded, then space maintenance is not necessary. Loss of deciduous molars is usually due to caries. If this is the case, long-term space maintenance in a caries-prone mouth may cause further harm.

Answer 3.8 A A  lateral cephalometric radiograph is a sagittal view of the patient’s skeletal and facial structures taken in a standardized manner, with the patient’s head placed in a cephalostat. The patient is positioned 2 m from the X-ray beam and less than 0.3 m from the film, with the Frankfort plane parallel to the floor and the head held in position by the ear rods of the cephalostat. The mid-sagittal plane of the patient is parallel with the plane of the film and the patient must bite together in undisplaced centric occlusion. The central X-ray beam is perpendicular to the sagittal plane and is centred to the middle of the film. If these distances are varied, then the magnification of the final image is distorted. There is always some magnification, but this should be standardized at 5–2%. Some operators prefer to position the patient using natural head posture rather than the Frankfort plane, as this may allow greater reproducibility for subsequent images of the same patient. B Lateral cephalometric radiographs supplement the clinical examination. They are used as a means of assessing the hard and soft tissue relationships of the patient’s facial structure. The upper and lower incisor and interincisor angulation is also assessed. Various analyses have been created which compare the patient’s values to normal values for the appropriate population. This enables the clinician to assess the severity of the patient’s malocclusion compared with the normal population group and aids treatment planning. In addition to diagnosis and treatment planning, cephalometry may be used to predict the likely growth pattern of the patient and may also be used as a research tool. C Care should be taken to minimize radiation dosage and ensure a good-quality film. As the image is taken, appropriate exposure and radiation dose settings should be used. Rare earth screens must be used to reduce the intensity of radiation needed to generate an image. An aluminium wedge between the patient and the X-ray tube will enhance the soft tissue profile by minimizing ‘burn out’ of the soft tissues. X-ray scatter can be reduced by placing a grid over the film cassette. However, this tends not to be needed with modern machines. Finally, careful developing techniques will maximize image quality, if film images are still in use. However, digital imaging has eliminated potential errors introduced during development.

Answer 3.9 A Anteroposteriorly, the skeletal relationship is severely class III, demonstrated by the ANB difference of  –2º. The maxillary mandibular planes angle is 5º, which is significantly reduced compared to the norm of 27º and shows that the patient’s skeletal relationship is reduced in the vertical plane. B The lower incisors are upright at 80º to the mandibular plane. The normal angulation is 93º +/– 5º. The upper incisors are very proclined at 33º relative to the norm of 09º +/– 5º. These normal values depend on which populations they are drawn from. In this case, Caucasian norms were appropriate, but this patient’s incisor angulations are

Treatment planning  127 extreme for any population group. The incisors have tipped beyond normal values in an attempt to overcome the severe class III skeletal relationship. This is an example of dentoalveolar compensation. C The overjet is reversed but the incisors are almost edge to edge; therefore, the reverse overjet is small. This is surprising in view of the severity of the underlying skeletal relationship and is because the incisors have attempted to compensate for this. D The value for SNA is 8.5º and this is essentially normal, showing that the maxilla is correctly positioned. Therefore, much of the skeletal III relationship can be attributed to a large mandible. E The severity of the skeletal relationship and the degree of dentoalveolar compensation already present indicates that a normal overjet and overbite will not be achievable by orthodontic means alone. Therefore, a combined orthodontic and orthognathic approach will be required if the patient wishes to have treatment.

Answer 3.0 A The incisor relationship is a class II division 2, where the lower incisors occlude palatal to the cingulum plateau of retroclined upper incisors. The overbite is increased. B The maxillary position is normal with an SNA of 8.5º. This suggests that much of the malocclusion is due to a small or retropositioned (retrognathic) mandible. C The upper incisors are retroclined at 99º but the lower incisors are slightly proclined at 98º. Therefore, there has been some dentoalveolar compensation for a moderately severe skeletal II relationship. In a class II division 2 type malocclusion, the overjet is usually within normal limits or reduced. Here, it is slightly increased at 6 mm (the norm for the population is 3 mm +/–  mm) due to the underlying anteroposterior skeletal discrepancy. In fact, the overjet should be even greater but has been minimized by the compensated incisor positions. This can lead to underestimation of the severity of the skeletal relationship and demonstrates that incisor angulation must be carefully considered at diagnosis.

Answer 3. A IOTN stands for the Index of Treatment Need and PAR stands for Peer Assessment Rating. B The difference between these indices is that the IOTN assesses the need for treatment whilst the PAR index is a means of measuring the improvement achieved by a course of orthodontic treatment. C The IOTN is divided into a dental health component (DHC), which is further subdivided into five categories, and an aesthetic component (AC), which has ten categories. The DHC is the main guide to determining treatment need. A Grade  has no need for treatment and a Grade 5 has the greatest need. Within this latter group are patients with cleft lip and/or palate, overjets greater than 9 mm, reverse overjets greater than 3.5 mm, missing teeth with restorative implications, supernumerary teeth, and impacted teeth (not third molars).

chapter four Pathology Answer 4. A This is a unilateral cleft lip and palate. B Clefting in humans occurs in approximately  in 700 live births, although this varies depending on race. In black races, there is a very low incidence, whereas in races from South East Asia, the incidence is much higher ( in 500 live births). The prevalence of unilateral cleft lip and palate within the cleft population varies between 6 and 25%, depending on the population examined. C The aetiology of clefting is not known. There is a genetic element, but this is not fully understood. There are familial tendencies and clefts of the palate are seen in a significant number of head and neck syndromes. Anti-epileptic drugs, such as phenytoin, are associated with clefting, as is the acne drug ‘Roaccutane’ (isotretinoin), which should be avoided during pregnancy. High doses of vitamin A have caused experimentally induced cleft palates in rodents. D The timing of palatal shelf elevation is critical. In humans, this occurs 6–8 weeks in utero. The elevation of the shelves happens remarkably quickly (usually within 24 hours) and depends on the shelves elevating above the tongue. This may require the tongue to drop. The mechanisms of shelf elevation which have been postulated are:

An increase in vascularity at the tips of the shelves This may increase their rigidity, causing them to elevate.

An increased accumulation of proteoglycans at the palatal shelf tips These molecules can absorb enormous amounts of water, increasing the turgidity and causing the shelves to elevate.

Contraction of collagen An ‘intrinsic’ force is thought to be generated and this may cause the shelves to elevate. The mechanism is not fully understood. After shelf elevation, formation of the palate is dependent upon removal of the medial edge epithelium (through apoptosis, programmed cell death, or epithelial mesenchymal transformation), mesenchyme flow, and development of myogenic and ostoegenic blastemata.

Answer 4.2 A Expression of clefting in humans may be divided as follows:

Clefts of the lip (unilateral or bilateral) The clefts are described as being complete if there is no union. Expression varies from a simple notching to a complete cleft. The alveolus may or may not be affected but cleft alveolus often occurs with clefting of the lip.

Pathology  129 Clefts of the palate These are divided into those of the hard or soft palate. Clefting in the soft palate can vary from a simple bifid uvula to a complete cleft. There may also be a submucous cleft where the hard palate is cleft but the soft palate is intact. This may first be detected by speech therapists, since these patients often have hypernasal speech. Clefts of the hard palate have variable expression. They are either complete or incomplete and may or may not involve the alveolus. Various classifications have been suggested. None have been universally accepted and so a written classification offers a great deal. B The surgical management of cleft lip and palate depends on the severity of the anomaly. Management is always a compromise between function and aesthetics. Surgery restores function, enabling the child to eat without nasal regurgitation and to speak more clearly. However, the scarring affects facial growth to a variable degree. In populations where people with unoperated clefts have survived into adulthood, facial growth is not normally compromised. The timings of surgical repair vary between hospital specialist units but the general timings for closure of various aspects of the cleft are as follows:

Lip closure at 3 months If the condition is bilateral, some surgeons repair one side and then the other, 4 weeks later. Others have preferred to close the lip (and in some cases, the lip and palate) at 24–48 hours after birth. There is no good evidence that this latter technique has any beneficial effects and the consequences of early scarring are likely to compromise normal facial growth. However, proponents of neonatal surgery suggest that it is of great psychological benefit for the parents of a cleft child.

The hard and soft palates are usually closed at 9 months Hard palate closure involves mobilizing palatal mucosa across the cleft. There is no repair of the bony defect. In some units, there has been a vogue for delayed closure of the hard palate until 7 or 8 years of age. This was found to result in very poor speech but good facial growth.

Alveolar bone grafting Grafting is carried out when the root of the permanent canine is two thirds formed (at 8–0 years of age). Autogenous bone (usually from the hip) is grafted into the alveolus. This provides bone into which the canine can erupt and it also stabilizes the major and minor segments of unilateral or bilateral clefts.

Orthognathic surgery Patients who have not experienced favourable facial growth may need orthognathic surgery once growth has ceased. This is usually around the age of 8 years, although midface osteogenic distraction may be performed at a younger age. C A typical cleft team comprises a specialist nurse, a consultant cleft surgeon (usually maxillofacial or plastic trained), a consultant orthodontist, a consultant maxillofacial surgeon, a consultant restorative dentist, a speech and language therapist, a clinical psychologist, and a paediatric dentist. D The orthodontist is involved at several stages:

Birth On the birth of the baby, the orthodontist or, more likely, a specialist cleft nurse, may give advice about feeding and introduce the family to the cleft team. Historically, some orthodontists have provided labial strapping and acrylic plates to try to bring the cleft segments closer together prior to primary closure (presurgical orthopaedics). However,

130 

Illustrated Questions in Orthodontics

this has not been found to be more beneficial than non-intervention and is not recommended practice.

Age 8– years Prior to alveolar bone grafting, it is common for the child to have orthodontic expansion to widen the cleft segments, correct crossbites, and create space in the cleft alveolus for surgical bone grafting. If teeth, such as upper central incisors, are impacted, interceptive orthodontics may be required to align them.

From age 2 Definitive orthodontics is left until the majority of the permanent teeth have erupted. If midface osteogenic distraction is planned, orthodontic involvement will be required in the mid–teenage years.

From age 7 If maxillary facial growth has been unsatisfactory (largely as a result of surgical scarring), the cleft patient will present with a relative mandibular prognathism (resulting in a class III skeletal base), and may opt for orthognathic correction. The orthodontist will provide pre- and post-surgical orthodontics and prepare the patient for any definitive restorative dental work required once surgery is complete.

Answer 4.3 A All types of cleft may be associated with abnormalities in the dentition. It is not uncommon to find a number of permanent teeth missing, particularly the second premolars. Paradoxically, supernumerary teeth may form in the cleft site. Presumably, the disruption of the dental lamina in the cleft site creates ‘satellites’ which produce tooth germs. The morphology of the teeth may also be altered and, frequently, the crowns are smaller in cleft children than in the non-cleft population. B It is vital that teeth are preserved in cleft patients. Therefore, a routine of regular dental care and good oral hygiene measures should be established from an early age. Appropriate fluoride supplementation is essential for the developing dentition and all first molars should be fissure sealed.

Answer 4.4 A This is a labial fraenum, which is attached in an abnormal, low position. B It can cause and maintain a diastema between the upper central incisors and prevent them being completely approximated. After orthodontic treatment, a fleshy fraenum can cause a diastema to reopen. It can also impede oral hygiene and may be considered unsightly. C In order for the teeth to be properly approximated, this fraenum needs to be surgically removed (fraenectomy). If removed prior to orthodontic treatment, full access to the surgical site is possible and the fibrous attachments to the interdental bone can be completely removed. However, it is also argued that the resultant scar tissue makes tooth movement difficult since teeth are being moved in fibrous tissue rather than bone. If it is removed after orthodontic treatment, the scarring may help maintain closure of the diastema. D If the palatal insertion of the fraenum blanches when the lip is lifted, surgery is recommended.

Pathology  131

Answer 4.5 A This is a midline supernumerary tooth, known as a mesiodens. B No. Supernumerary teeth usually remain unerupted and are discovered radiographically. C The incidence of supernumerary teeth in the permanent dentition is 2% and % in the deciduous dentition. D Incisors can be displaced from their normal position or fail to erupt due to the presence of a supernumerary. Alternatively, the supernumerary tooth may erupt, needing to be extracted, or it may cause crowding. In general, supernumerary teeth should be removed prior to orthodontic treatment, in order to avoid the risk of potential root resorption of the permanent teeth. However, if they are well away from the teeth to be moved, they can be left in place and monitored.

Answer 4.6 A The lower left second molar is unerupted, unlike the right second molar that has fully erupted. Asymmetrical eruption patterns should give rise to concern and be investigated, especially if one tooth is fully erupted and the contralateral tooth is not, as seen here. B A dental history should reveal if the tooth has been previously extracted and a radiograph should be used to confirm the presence of the tooth and its position. Surgical removal of any obstruction will be necessary and the patient should be kept under regular review to ensure that the molar erupts. If it fails to erupt spontaneously, then active intervention with fixed orthodontic appliances will be needed. C Dentigerous cyst formation around the crown and impaction by the third molar tooth germ are the most common causes of failure of eruption of the second molar. Patients with posterior crowding (i.e. in the molar area) may have a second molar impacted mesially against the first molar. However, this particular case is uncrowded and was found to have a dentigerous cyst associated with the second molar.

Answer 4.7 A This is a talon cusp, a rare developmental mass of hard tissue projecting from the cingulum. It most commonly affects upper lateral incisors and is more common in males than females. In addition to the aesthetic impact, the talon cusp can cause tongue irritation and there is a risk it may fracture, causing pulpal exposure. B It is likely that the pulp horn extends into the talon cusp. Therefore, if the talon cusp is to be reduced, it is possible to cause pulpal exposure. Two approaches can be considered: elective root canal treatment to devitalize the tooth, reduce it in size, and restore it, or progressive reduction of the talon cusp with application of fluoride varnish over several visits. The presenting size of the talon cusp and sensitivity will influence choice of management. C Unless the upper central incisor is restored to its normal anatomy, it will be impossible to fully reduce the overjet.

Answer 4.8 A This is a geminated upper right central incisor. This patient presented with an unerupted upper right central incisor and had it exposed and bonded with a gold chain procedure. When it erupted, it was evident that it was geminated. A normal upper right lateral incisor was developing.

132 

Illustrated Questions in Orthodontics

B Tooth gemination occurs more frequently in the deciduous dentition than the permanent dentition, with figures of 0.6% and 0.% respectively. C Geminated teeth occur when two teeth develop from a single tooth bud, as opposed to fusion where two adjacent tooth buds unite. Fused teeth may have two independent pulp chambers and root canals evident, in which case it may be possible to divide the fused tooth into two. However, geminated teeth normally have a single root canal, in which case the crown is effectively macrodont and extraction, as part of a restorative treatment plan, may be a more appropriate option. D If the number of teeth is normal, a macrodont crown causes a tooth size discrepancy and will result in an increased overjet, if all teeth are to be aligned within the arch. However, in this case, it was decided to extract the geminated tooth and reduce the anterior space to provide a normal sized restorative replacement for the upper right central incisor.

Answer 4.9 A This is an occlusal view of a child in the early mixed dentition, with a well aligned arch, apart from mild irregularity of the lower central incisors. The lower left first molar is fissure sealed, but the lower right first molar is mottled and restored with glass ionomer cement (a glass ionomer ‘bandage’). This condition is called molar incisor hypomineralization (MIH) because the first molars and central incisors are commonly affected. B The cause is not understood, but some form of trauma at the time of enamel formation is thought to be associated, e.g. febrile illness, respiratory problems, birth by Caesarian section, have all been proposed as possible causes. C The deficient enamel can cause the teeth to be sensitive to cold and tooth brushing, and to be unaesthetic. D Mild cases can be managed with the application of fluoride and fissure sealants when the tooth is fully erupted. Badly affected molars may need to be extracted and incisors may need to be veneered. E First molars are ideally extracted at the time of root bifurcation of the lower second molars, i.e. around the age of 9–0 years old. Ideally, the tooth germ of third molars will be visible radiographically. This will give the best chance of the lower second molars erupting alongside the second premolars, i.e. replacing the position of the first molars.

Answer 4.0 A The bone/periodontal interface demonstrates bone-forming activity. The bone is lined by osteoblasts and there is relatively recently formed bone subjacent to these cells. B The periodontal ligament has large blood vessels close to the bone. There are fewer vessels adjacent to the cementum. Some have postulated that anti-angiogenesis factors produced by the cementum may be responsible for this. C Bone is being laid down here, not resorbed. Therefore, the tooth is moving away from this area.

Answer 4. A This image demonstrates hyalinization, caused by excessive force being applied to the tooth, which compresses the blood vessels in the periodontal ligament.

Pathology  133 B Once the excessive force has been removed, the periodontal ligament will recover. The blood vessels will re-establish themselves and attempts at repair of any cemental damage will be made. C Excessive pressure causes pain and undermining bone resorption (which may slow down tooth movement). The cementum and root are thought to undergo more resorption once they have been exposed to these types of forces.

Answer 4.2 A The bone at the periodontal interface is being resorbed by multinucleated osteoclasts, some of which can be seen lying in their Howship’s lacunae. B The tooth is moving towards the area of bone resorption, i.e. towards this periodontal ligament. C The origin of the osteoclast is thought to be fusion of circulating monocytes.

chapter five Removable appliances Answer 5. A The fundamental difference between removable and fixed appliances is that removable appliances tip teeth whilst fixed appliances can move them bodily. Removable appliances tip the tooth about its centre of resistance, considered to be the root centroid of single-rooted teeth (40% from the root apex) or the trifurcation of multirooted teeth. Removable appliances can also hold blocks of teeth, preventing or allowing selective tooth eruption. B The force delivered by a wire spring is controlled by several variables according to the equation: F α Edr4 l3 where F = force, E = elastic modulus of the wire, d = deflection of the spring on activation, r = wire radius, and l = length of the wire. Therefore, the stiffness, thickness, and length of the wire and the degree of activation affect the force delivered by the appliance. The length can be increased by the incorporation of loops. The optimal force is considered to be 25–40 grams for single-rooted teeth and 40–60 grams for multirooted teeth. C Removable appliances remain useful in current orthodontic practice for a number of tasks: i tipping teeth, e.g. anterior crossbite correction, arch expansion, molar distalization, conversion of a class II division 2 to a class II division  malocclusion prior to functional appliance therapy ii holding blocks of teeth, e.g. overbite control, space maintenance, retainers iii habit breakers. D The advantages of removable appliances over fixed appliances include: i greater effectiveness at achieving certain tooth movements, e.g. overbite reduction in a growing patient ii good oral hygiene is easier to maintain iii less chance of root resorption iv minimal chair-side time needed for adjustments v straightforward anchorage management if the appliance is worn well. The disadvantages of removable appliances are: i success relies on the patient co-operating and wearing the appliance ii only simple tooth movements can be achieved, compared to the complex bodily tooth movement achieved by fixed appliances iii only a limited number of tasks can be achieved with one appliance iv speech is adversely affected and salivation increased in the short term v an impression is required for construction, which is rarely popular vi they require laboratory backup and time.

Removable appliances  135

Answer 5.2 A ARAB stands for: Active components; Retentive components; Anchorage; Baseplate. B Several designs would work equally well. Here is one possible design: i double cantilever spring (Z springs) in 0.5 mm stainless steel wire palatal to the upper left central incisor ii double Adams cribs on the upper first and upper second deciduous molars (0.7 mm stainless steel wire) iii posterior bite capping iv baseplate.

General feedback Retention in the middle of the dental arch (i.e. the double crib) is needed because activation of the Z springs anteriorly will displace the appliance downwards. If only the first molars were cribbed, the appliance would be very easily displaced. C Retention is not necessary after treatment providing that the overbite is average or increased, as it will prevent the incisors from relapsing.

Answer 5.3 A This appliance is being used to expand the upper arch and to procline the upper lateral incisors out of crossbite. B Clinicians vary in their instructions regarding the use of the expansion screw. However, a common instruction is that the screw should be turned by ¼ turn, twice a week, i.e. the screw is turned until the next hole becomes visible. This provides expansion at a rate of 0.25 mm per ¼ turn. Some clinicians start with turning the screw ¼ turn, once a week. C The disadvantage of this method of arch expansion is that it relies totally on patient co-operation, firstly in turning the screw and, secondly, in turning it in the correct direction. This can be helped by the incorporation of an arrow in the baseplate to show the direction in which the screw should be turned. D Other methods of upper arch expansion include: i Coffin spring in a removable appliance ii quadhelix iii rapid maxillary expansion iv expansion with an archwire v surgical expansion of the maxilla. E The posterior bite capping has two functions here. It is opening the bite, enabling the upper lateral incisors to procline, and it is freeing occlusal contacts, enabling the maxillary arch to expand. The patient and parent should be warned that the bite plane will chip in use but they do not need to return to the surgery unless the appliance breaks. F Lifting the T springs gently away from the baseplate activates them. Too much activation will displace the appliance. Alternatively, Z springs in 0.5 mm stainless steel wire could be used. These are slightly more difficult for the patient to seat but have a longer range of activation than the T springs. G The expansion achieved will be the most prone to relapse. The upper lateral incisors should be held in the correct position by the overbite, providing it is sufficiently deep. H This appliance is designed as follows: i T springs in 0.5 mm stainless wire behind the upper lateral incisors ii midline expansion screw

136 

Illustrated Questions in Orthodontics

iii Adams cribs in 0.7 mm stainless steel wire on the upper first molars and Jacksons cribs in 0.6 mm stainless steel wire on the upper first deciduous molars iv posterior bite capping v baseplate.

Answer 5.4 A The purpose of this appliance is to discourage a digit-sucking habit. It is known as a hayrake appliance. B The problem with this appliance is that it is removable and the patient can remove it in order to suck their digit! Therefore, success is limited by patient co-operation. C The posterior teeth are capped in order to encourage greater eruption of the incisors (which have been intruded by the digit habit) relative to the buccal segment teeth. D The tubes soldered to the bridge of the Adams cribs are used for the attachment of a facebow, so that high pull headgear can be fitted to intrude the upper buccal segments, thereby reducing the anterior open bite.

Answer 5.5 A Five removable appliances designs: i URA to distalize the upper first molars in conjunction with headgear attached to molar bands

Figure 5.5a



Note: self-guarding palatal finger springs mesial to the upper first molars in 0.6 mm stainless steel wire; flat anterior bite plane; Adams cribs in 0.7 mm stainless steel wire on the upper first premolars; Southend clasp in 0.7 mm stainless steel wire on the upper central incisors; baseplate. ii a clip-over bite plane to be used with fixed appliances

Removable appliances  137

Figure 5.5b



Note: flat anterior bite plane; clip-over clasps in 0.7 mm stainless steel wire on the upper first molars; torquing type spurs in 0.5 or 0.6 mm stainless steel wire on the upper central incisors, incisal to the brackets; baseplate. iii a space maintainer for a missing upper left central incisor

Figure 5.5c

Note: wire stops mesial to the upper left lateral incisor and upper right central incisor in 0.7 mm soft stainless steel wire; Adams cribs in 0.7 mm stainless steel wire on the upper first molars; baseplate.

138 

Illustrated Questions in Orthodontics

iv unilateral screw plate for crossbite correction of the upper right buccal segment

Figure 5.5d



Note:  midline expansion screw; Adams cribs in 0.7  mm stainless steel wire on the upper first molars and upper first premolars; Southend clasp in 0.7 mm stainless steel wire on the upper central incisors; posterior bite capping; baseplate, split between the upper right canine and first premolar. v unilateral screw plate for distalization of the upper left buccal segment

Figure 5.5e

Removable appliances  139

Note: expansion screw orientated anteroposteriorly; Adams cribs in 0.7 mm stainless steel wire on the upper first molars and upper first premolars; Southend clasp in 0.7 mm stainless steel wire on the upper central incisors; posterior bite capping; baseplate, split between the upper left canine and first premolar. B In accordance with Newton’s third law of motion, all forces have a resultant equivalent force which acts in the opposite direction. Therefore, forces used to distalize teeth will also encourage the unwanted proclination of teeth anterior to the point of force application. This is called anchorage loss. If the overjet increases during treatment, this indicates anchorage loss, and overjet measurement at each visit is a good means of monitoring anchorage management. Anchorage can be controlled by the addition of headgear worn at nights.

Answer 5.6 A This is the Invisalign® appliance. It is an orthodontic appliance which requires a succession of plastic aligners to be worn over a period of time. Each aligner delivers a subtle progressive tooth movement over a two-week period until the treatment goals are achieved. On completion of the first phase of treatment, it is common that a second cycle of treatment, called refinement, is provided. B It is suitable for the treatment of patients with mild malocclusions such as lower incisor imbrications, closure of minor spaces, and minor rotations. It can be used to treat patients with more difficult malocclusions, providing that the treatment goals are modified accordingly. Whilst tooth intrusion is achievable, it is not good at extruding teeth. C Silicone-based impressions of both arches are taken and sent to the commercial laboratory, where they are scanned. Alternatively, a scan of the dental arches may be submitted digitally. Radiographs and photographs are submitted either as hard copies with the impressions or as digital images online. A treatment planning form also needs to be submitted, with the plan based on conventional orthodontic treatment principles. A 3D virtual set-up of the proposed treatment plan, called a Clincheck, is prepared by the Invisalign® team and viewed via the Invisalign® website. This is modified by the treating clinician until the proposed treatment plan is satisfactory in outcome and viability. At this point, the orthodontist instructs manufacture and the aligners are laser engineered from medical quality plastic, before despatch to the clinician. D Composite attachments to the enamel tooth surfaces are designed to enhance tooth movements. These are added by using an attachment template, also created from plastic. Additionally, precision cuts in the aligners can be requested to allow the application of elastics and buttons for interarch elastic traction use. E There is little doubt that Invisalign® treatment can move teeth but the appliance intrinsically struggles with tooth movement in all three dimensions. It is popular with patients but, in the hands of inexperienced clinicians, can create difficulties which require correction with conventional orthodontic approaches. The Invisalign® default maximum speed is approximately 0.25 mm of translation (approximately 0.5 mm a month if each aligner is worn for 2 weeks) and  degree rotation per stage. F Aligners should be worn for 22 hours per day, for two weeks, before progressing to the next aligner. They should be removed for eating and drinking anything other than water and cleaned using toothpaste and cold water. G Class III traction is being used to correct an anterior crossbite. The elastic is attached to a standard molar bracket and a cut-out in the aligner in the canine region. A button could be used instead of a bracket.

140 

Illustrated Questions in Orthodontics

Answer 5.7 A This is the Somnowell™ appliance, which is one of a variety of mandibular advancement splints/appliances. B Commonly, it is used to manage sleep disorders such as snoring and obstructive sleep apnoea. However, it can also be used for temporomandibular joint dysfunction and bruxism. C The mandible is postured forwards, bringing the tongue with it, which reduces the contact between the tongue, soft palate, and pharyngeal wall. This increases the size of the pharyngeal airway. D Good-quality impressions are taken, ideally in polyvinylsilicone impression material, together with a postured bite and a facebow recording. It is important to ensure the condyles are not displaced laterally when the bite registration is taken. The Somnowell™ appliance is constructed in cast cobalt chrome in a dental laboratory. E It is important that the patient is dentally fit, with all dental work completed prior to the construction of the Somnowell™ appliance. The patient must be warned that their bite may be slightly disturbed on waking and may require some mild manipulation to return the occlusion to normal. F Continuous positive airway pressure (CPAP) has been regarded as the most effective way of managing obstructive sleep apnoea. This involves the patient wearing a face mask through which air is pushed, under positive pressure, into the upper airway. However, the machinery is noisy and the face mask can be uncomfortable to wear, so compliance with CPAP can be low, even though it is a successful technique.

chapter six Functional appliances Answer 6. A This functional appliance is a medium opening activator (MOA). B It is retained by the Adams cribs on the upper first molars and the acrylic capping the lower incisors. C Traditionally, functional appliances have been considered ideal for the treatment of a well-aligned class II division  malocclusion, with a large overjet and deep overbite, in a growing patient. This particular functional appliance is suited to patients with a deep overbite, low maxillary–mandibular planes angle, and reduced lower facial height. The lower acrylic extends lingually only in the lower labial segment area and the buccal segments are not capped. These features allow the lower molars to erupt as the overjet is reduced, aiding overbite reduction and increasing the lower facial height. Additionally, it is useful in the mixed dentition, as the loss of deciduous teeth is unlikely to affect the fit of the appliance. D Several limitations affect the use of an MOA. As the appliance is essentially a monobloc, upper arch expansion cannot be carried out at the same time as overjet reduction. Therefore, arch expansion must be achieved with an appliance prior to fitting the MOA. Unlike the Twin Block appliance, the MOA must be removed for eating, and clear speech is more difficult. As a result, it is common to ask patients to wear the appliance on a part-time basis only.

Answer 6.2 A This functional appliance is known as Clark’s Twin Block appliance. B It is predominantly used to correct a large overjet and deep overbite in a patient with a class  II division  malocclusion and normal or increased lower anterior face height. Although useful in the late mixed dentition to reduce the overjet prior to definitive orthodontic treatment, the loss of deciduous teeth during treatment can interfere with the fit of the appliance. Classically, functional appliances are used in class II division  cases, where the skeletal discrepancy is mild, crowding is minimal, and the dental bases are long. They may also be used in crowded cases and class II division 2 cases in which the upper incisors can be simultaneously proclined with a spring in the upper removable appliance (URA). Alternatively, a URA to procline and expand the upper arch can be used first. C Functional appliances act by using orofacial muscular forces. Their primary effect is to cause dentoalveolar change, i.e. teeth tip. However, there is some evidence to suggest that condylar repositioning and a minor element of growth modification (maxillary restraint and redirection of mandibular growth) also help to reduce the overjet. Once the overjet has been reduced, the blocks of the Twin Block appliance are trimmed to allow posterior teeth to erupt, thus reducing the overbite. D Twin Blocks have several advantages over other functional appliances. They can be worn 24 hours a day, even during eating. Maxillary expansion with a midline expansion screw

Functional appliances  143 can be achieved concurrently with overjet reduction. Speech is normalized more rapidly with the wearing of the appliance 24 hours a day. E Headgear can be attached to soldered extraoral traction (EOT) tubes on the bridges of the Adams cribs or EOT tubes embedded in the acrylic. The use of EOT will help reduce the overjet and aid restraint of maxillary growth in the anteroposterior and vertical planes.

Answer 6.3 A This is a Frankel functional appliance. In total, there are four variants of the Frankel appliance to treat class  I, II, and III cases and cases with anterior open bite or bimaxillary proclination. B The Frankel appliance was invented by Dr Frankel in former East Germany. Limited access to fixed appliance components after the Second World War hindered their use and Dr Frankel created appliances that could be more readily manufactured using available materials. He termed his group of appliances ‘function regulators’. C Unlike the other functional appliances that use a postured bite to access orofacial muscular forces, the Frankel appliance has carefully positioned buccal and labial acrylic shields. These alter the balance of muscular forces by lifting the soft tissues away from the teeth, enabling them to tip into the space created. Therefore, the correction of a class III malocclusion with this appliance is by dentoalveolar means, not by skeletal modification. The Frankel 2 appliance, used to correct class II malocclusions, has both lingual and labial acrylic pads in the lower labial segment, which encourage the mandible to be held in a postured position. The buccal shields are extended to fill the buccal sulcus, which Dr Frankel theorized would induce periosteal stretch, therefore stimulating bone formation and altering lip behaviour. There is no evidence to support this theory of skeletal correction. D This appliance is particularly useful for orthodontic correction in the mixed dentition. This is because the appliance is primarily soft tissue borne, rather than tooth borne, and, therefore, wear is minimally affected by exfoliation of the primary dentition. E Alternative methods of correcting a developing class III malocclusion include the reverse Twin Block functional appliance, in which the bite blocks displace the mandibular condyles backwards in the glenoid fossae. Chin cup therapy also attempts to limit mandibular development, but studies have shown that its effects are limited to causing the lower incisors to retrocline, without an effect on mandibular growth. Finally, protraction headgear, often in conjunction with maxillary expansion, can be used to pull the maxillary dentition forwards. Whilst there is little evidence to show that growth modification can be achieved for skeletal II cases, there is even less evidence to support growth modification in skeletal III cases. The majority of change is achieved by dentoalveolar means and may be transient whilst active facial growth is continuing. Moderate to severe skeletal III cases may only be correctable with orthognathic surgery.

Answer 6.4 A This appliance is a clip-over Twin Block. B It is useful for the correction of a large overjet during fixed appliance treatment, since it may be worn concurrently. This is helpful if a patient has a relapse of overjet reduction during the fixed appliance phase of treatment following a previous course of functional appliance treatment, or if it is desirable to use the two appliances simultaneously as a means of trying to reduce the length of treatment. C At the outset, the patient should have completed orthodontic alignment and be wearing heavy archwires (ideally, 0.09” x 0.025” stainless steel). Alginate impressions are taken

144 

Illustrated Questions in Orthodontics

of the dental arches without the archwires in situ, together with a postured bite. The clip-over Twin Block is constructed on articulated study models in the orthodontic laboratory. It is retained using ball clasps between the premolars and molars, and has acrylic bite blocks as the active component, which posture the mandible forwards.

Answer 6.5 A This is a Herbst appliance. B It is a fixed functional appliance, attached to upper first molars and lower first premolars. The active component is the sliding piston, which causes the mandible to protrude. C The principal advantage is that the appliance cannot be removed by the patient, therefore compliance is ensured. D It cannot be removed for eating and there is no means of the patient getting used to the appliance gradually. E This patient has a reverse overjet because the active pistons are pushing the mandible forwards. The magnitude of the reverse overjet will increase as the appliance becomes effective and can be measured to assess progress.

chapter seven Fixed appliances Answer 7. A This fixed appliance is a straightwire appliance. First developed in the 980s, it is the successor of the standard edgewise fixed appliance system. B The fundamental difference between the standard edgewise and the straightwire appliance is that the final tip, torque, and buccolingual positions of the tooth are built into the bracket of the straightwire appliance by varying the slot angulation, torque, and base thickness. Therefore, if the bracket is correctly positioned on the tooth and a maximal dimension rectangular wire is used, the prescribed position should be automatically achieved. Standard edgewise brackets have no such tooth positioning features; hence, tooth position is dependent on wire bending. In this picture, the wire is not bent (except as an archform), so it is the straightwire appliance. C The principle advantage of the straightwire appliance is that treatment mechanics are much simpler than with the standard edgewise appliance. Using the straightwire appliance, final tooth position is dictated by bracket positioning rather than wire bending. As the wire remains flat, tooth movement is achieved by sliding mechanics rather than complicated closing loops. Other than modifying the archform, wire bending is generally only needed to finish the case to a high standard. Treatment is technically easier and chair-side time is much reduced. D The principal components of fixed appliances can be considered as those used as a means of attachment to the teeth, such as bands and brackets, and active components which achieve tooth movement, such as wires and elastomeric products. Orthodontic bands are commonly cemented with glass ionomer cement, whilst brackets are bonded to teeth with composite cement. Glass ionomer cement for bracket bonding has been proposed as a means of reducing decalcification. The bond strength of glass ionomer is approximately half that of composite. Archwires are ligated to the brackets with either elastomeric modules or stainless steel wire. Finally, elastic bands, elastomeric chain, and various types of metal coil springs are used for tooth movement. These are generally attached to hooks on the appliance, usually on the bands or archwires. E These elastics are attached posteriorly in the mandible (to hooks on the first molar bands) and stretch anteriorly to hooks in the maxillary lateral incisor region. This type of elastic wear is called class II traction.

Answer 7.2 A This is the Tip Edge fixed appliance system, which has evolved from the Begg appliance. B Unlike the straightwire (and standard edgewise) fixed appliance systems which move teeth bodily along the archwire to planned positions, the Tip Edge appliance achieves tooth movement by causing the crowns of the teeth to tip along the archwire, often into extraction spaces, before uprighting the roots as a secondary procedure. This method of tooth movement is less demanding on anchorage than bodily tooth movement.

Fixed appliances  147 C Tip Edge treatment is divided into three stages. Stage  achieves tooth alignment and reduction of the overbite and overjet. During this stage, only the labial segment teeth and first molars are involved in the appliance. At the beginning of Stage 2, the remaining teeth are incorporated into the appliance and extraction spaces are closed. Stage 3 is primarily concerned with root uprighting and achieving the correct torque. The crowns of the teeth are in the correct positions, but the roots remain tipped. Using rectangular wire and torquing springs (sidewinders) the roots are moved into the correct position. D Light class  II traction (elastic bands attached between the mandibular molars and the upper lateral incisor/canine region) worn full time, especially during meals, provides much of the mechanical force to achieve tooth movement. Class II traction causes the mandibular molars to extrude and roll lingually, both unwanted effects. This can be limited by tip back bends in the lower archwire, which should also be expanded considerably. Maxillary incisors can also be extruded by class II traction, but this is minimized by tip back bends in the upper archwire which carry the archwire high into the labial sulcus anteriorly. Thus, the intrusive effects of the archwire counteract the extrusive effects of the elastic.

Answer 7.3 A This is a photograph of an upper fixed appliance with ceramic brackets. The upper left permanent canine is unerupted and a steel ligature (laceback) is protecting the archwire in the canine space from occlusal forces. B Ceramic brackets have very good aesthetics, although this effect is lessened as the archwire increases in diameter and the elastomeric modules stain. Stainless steel ligature ties are an aesthetic alternative to modules. Unfortunately, ceramic brackets have some disadvantages. They increase friction between the bracket and the archwire, reducing the effectiveness of sliding mechanics. Ceramic brackets are harder than enamel and can cause enamel abrasion. For this reason, they are not recommended for use on the lower incisors in patients with an increased overbite, as the overbite may cause wear to the upper incisal edges. Early types of bracket had excessive bond strength to enamel, causing enamel fractures at debond. Even with modifications to bonding materials and the bracket base, debond can be difficult and the brackets may need to be removed with a high-speed hand piece. During use, tie wings are particularly prone to fracture. Finally, ceramic brackets are expensive relative to stainless steel brackets. C Some brands of bracket incorporate a stainless steel slot to reduce friction, whilst the bracket base and cement are modified to reduce bond strength to enamel. A  careful debonding technique is required and some brackets have specific debonding pliers, designed to aid easy bracket removal.

Answer 7.4 A These brackets are self-ligating brackets. Specifically, these are Innovation R™ brackets, where the archwire is engaged with the bracket clip. B Unlike conventional brackets, the means of archwire engagement is an integral part of the bracket design, either with a passive clip (e.g. Innovation R™, Damon™ brackets) or an active clip (e.g. Smartclip™). Conventional brackets use elastomeric modules or stainless steel ligatures to hold the archwire into the bracket slot. C There are many different brands of self-ligating brackets on the market, including those used in lingual orthodontics. The DamonTM bracket (Ormco) is probably the most widely known, due to extensive marketing. However, other popular brands include Innovation RTM (TOC) and SmartclipTM (3M).

148 

Illustrated Questions in Orthodontics

D Significantly reduced friction between the archwire and the bracket slot and the transmission of lighter forces is the principal advantage of these types of bracket compared to conventional brackets. However, accurate bracket placement is more difficult, as is finishing to a high standard. Many of the claims made for self-ligating systems such as less pain, shorter treatment, quicker alignment, and a different biological response in tooth movement have not been substantiated E Some orthodontists have completely changed their practice to use self-ligating brackets as a matter of routine. Self-ligating brackets are purported to be useful in the management of patients with high anchorage demands, severely excluded teeth, and those needing large lateral tooth movements. For example, space opening for excluded teeth and alignment of severely crowded teeth is thought to be easier than with conventional brackets.

Answer 7.5 A This is the lingual orthodontic technique, whereby the braces are fitted on the lingual surfaces of the teeth. Specifically, this type of brace is called the Incognito™ appliance, a bespoke pre-programmed bracket system cast from dental gold alloy, with customized archwires, manufactured by 3M Unitek. B Improved aesthetics, combined with the performance of a fixed appliance system, are the advantages of this system. However, compared to a labial appliance system, it is difficult for the orthodontist to adjust and harder for the patient to keep clean. The patient will need to adapt to the appliance intruding on the tongue space, which interferes with speech transiently and can cause ulceration in the early stages of treatment.

General feedback Unlike labial brackets, the lingual bracket position is some distance from the labial tooth surface, which is the visible part of the smile. This can cause torque discrepancies to arise and is hard to correct. However, the flat profile of this type of bracket goes some way to overcoming this difficulty. C Silicone-based material impressions are taken of the upper and lower jaws, requiring accurate detail of the lingual tooth surfaces, together with a bite registration. These are sent, with a treatment plan, to the orthodontic laboratory. The appliances are returned inside an indirect bonding tray for fitting in the dental surgery. One jaw (commonly, the lower) is bonded first, so that the patient can become used to wearing the appliance in stages. Dental extractions are carried out after the appliance is fitted, but prior to archwire placement, in order to ensure the indirect bonding tray will fit. Unlike conventional labial braces, it is difficult to use direct bonding because of problems in accessing the lingual surfaces of the teeth to place brackets accurately. D The patient must be advised on how to clean the brace, what types of food to eat, and to expect some discomfort and speech disturbance. The placement of wax (from below the bracket base and squeezed around the archwire, so it does not become dislodged) should be demonstrated. The patient should be advised to wear the wax for at least the initial 2 weeks, particularly at night, to avoid tongue ulceration.

Answer 7.6 A This is an archwire constructed with closing T loops. This was a common means of closing space when standard edgewise treatment mechanics were used. Although this is a straightwire appliance, sometimes space closure using conventional sliding mechanics is so slow that it is necessary to resort to bending a wire with closing loops.

Fixed appliances  149 B The wire is pulled through the bracket slot on the first molars until the vertical legs on the T loops are open maximally and bent 90° behind the first molars. The T loops return to their intial position, bringing the buccal segment teeth with them. In this illustration, the T loops are passive.

Answer 7.7 A B

An ideal orthodontic archwire should have: i high strength to withstand occlusal forces ii low stiffness so that it can be engaged into a bracket iii a high range of deflection (the distance the wire bends elastically before it is deformed) iv formability so that it can be bent into hooks v the ability to accept solder so it can have hooks attached vi as little friction as possible vii biocompatibility viii stability in the oral environment ix low cost. Increasing the cross-section of a wire increases its strength and stiffness, but reduces its range of deflection. Therefore, thicker wires are used as working wires, along which aligned teeth can slide. They are not used for alignment, unless loops are incorporated to alter the length and properties of the wire. C Increasing the length of a wire by the incorporation of loops reduces the force applied to the teeth.

Answer 7.8 A This appliance is a quadhelix. B It is used for expansion of maxillary teeth. It can also be activated to achieve molar derotation. C The quadhelix should be activated so that when held over the study cast it is expanded relative to the maxillary arch by approximately half a molar tooth width on each side. The palatal arms will then extend along the occlusal surfaces of the teeth. A careful measurement of both the passive and active dimensions should be recorded so that an indication of the amount of activity remaining in the appliance can be ascertained without it needing to be removed from the mouth at each visit. The molar bands are filled with band cement and the quadhelix is contracted momentarily to its passive dimension whilst cemented in place. D The palatal cusp of the first molar tends to drop during expansion and can prop the bite open. This is a problem in cases with reduced overbite. Other problems include tongue irritation, the palatal mucosa may become hyperplastic, and the anterior coils may become embedded. Teeth can become excessively mobile. Band cement failure can lead to the appliance becoming dislodged. E Alternatives to the quadhelix include an upper removable appliance with a midline expansion screw or Coffin spring, rapid maxillary expansion (RME) with a hyrax screw, or an expanded upper archwire in a fixed appliance.

Answer 7.9 A This appliance is a rapid maxillary expander. B The screw should be turned twice daily, which should provide expansion at a rate of 0.5 mm per day, and the patient should be reviewed about 0 days after fitting.

150 

Illustrated Questions in Orthodontics

C The patient should be warned that a midline diastema will open between their maxillary central incisors. D This appliance is used to open the midpalatal suture, across which new bone then forms. Therefore, this appliance has an orthopaedic effect, inducing bone formation and a dentoalveolar effect by displacing teeth buccally. The other expansion appliances have limited orthopaedic effects and work primarily by tipping maxillary buccal segment teeth buccally.

Answer 7.0 A This appliance is a nickel titanium molar derotator and is derotating the upper first molars distally about the palatal root. It is a proprietary removable appliance and inserts into palatal sheaths on the first molar bands. It is compressed on insertion and returns to its original dimension, causing the molars to derotate as it does so. B The most likely cause of the upper first molars being rotated is premature loss of the upper second deciduous molars. In this instance, the upper right second premolar was palatally impacted and the upper left second premolar was palatally displaced. The first molars were being derotated in order to create space to accommodate the second premolars. This patient had a class III malocclusion and it was desirable to avoid upper arch extractions, otherwise the upper second premolars could simply have been extracted. C Alternatively, a transpalatal arch with a distal facing loop that could be progressively opened could also have been used.

Answer 7. A This is a hayrake appliance and is used to stop a persistent digit-sucking habit. The advantage of this style over the one shown in Figure 5.4 (Section , Chapter 5, Question 5.4) is that it cannot be removed by the patient. B Orthodontic bands are selected to fit the upper first molars and an alginate impression is taken. The hayrake appliance is constructed by an orthodontic laboratory technician and then cemented in place by the clinician.

chapter eight Anchorage Answer 8. A This appliance is a lingual arch. B Its function is to preserve anchorage in the lower arch so that all space gained from loss of deciduous teeth or from extractions of permanent teeth can be used as planned. Indications for a lingual arch include: i severe lower arch crowding requiring all the extraction space for alignment ii to preserve space after the loss of deciduous molars iii to aid correction of a lower centreline shift iv to hold space for the eruption or alignment of impacted teeth or teeth displaced from the arch. C Three visits are required to fabricate and fit a lingual arch. Firstly, molar separation with elastomeric separators or brass wire is needed for a week. Following molar band selection, an alginate impression is taken of the lower arch with the bands in situ. The bands are replaced inside the impression (and can be waxed in place) and sent to the laboratory. A stone model is poured and the lingual arch is fabricated. Separation space is maintained with brass wire or elastomeric separators until the following visit. Finally, the appliance is cemented in place. D The molar bands may displace within the impression and if not correctly repositioned, this will affect the fit of the appliance. E It is possible to fit headgear to the lower arch but this is even less popular than conventional headgear! A lower removable appliance could be considered but success is dependent on patient compliance and they are also poorly tolerated. A lip bumper attached to the lower molars is a further means of reinforcing lower arch anchorage. Finally, temporary orthodontic implants (also known as temporary anchorage devices, TADs) may also be considered.

Answer 8.2 A This appliance is called headgear. B Headgear use may be considered for both skeletal and dental correction. Vertical and anteroposterior maxillary development may be restrained by headgear use, providing it is worn for an adequate period of time. More commonly, headgear is used to control tooth position, e.g. for anchorage support in the upper arch, distalization of upper molars to correct a class  II buccal segment relationship, overjet reduction, intrusion of upper buccal segments and/or incisors. The combination of upper molar intrusion and maxillary growth restraint helps to correct a class II skeletal discrepancy. Skeletal change with headgear wear is minor and headgear must be worn over a long period of time (years) to see worthwhile change. Protraction headgear (not shown) can bring the maxillary dentition forwards and may be used for dental correction of a class III malocclusion or to aid anchorage loss, sometimes required in the management of hypodontia cases.

Anchorage  153 C The component parts of this headgear system are a headcap, facebow, force delivery modules, and safety strap. D The amount and duration of force application depends on the purpose for which headgear is being worn. For anchorage management, 250 grams force applied per side, with wear of 8–0 hours per day is sufficient. However, to achieve tooth movement or growth modification, a force of 500 grams per side for 2–4 hours per day is required. E Injury sustained whilst wearing headgear is rare but has been reported and includes a penetrating eye injury from a facebow, which resulted in blindness. Therefore, safety is very important and headgear should be worn with appropriate safety mechanisms, e.g. a safety strap, snap away headgear, and safety locking facebows.

Answer 8.3 A The molar relationship is Angle’s class  I  but the premolars are slightly class  II and the canines are a ½ unit class II. B There are a number of possible explanations for this malocclusion. There is a premolar missing from each arch. This may be due to the developmental absence of second premolars, but it is more likely that the first premolars were removed to relieve crowding. A final class I canine relationship was not achieved, which has limited full correction of the overjet. This poor result may be due to inadequate patient co-operation, resulting in failure to fulfil treatment objectives, or to inadequate anchorage management during active appliance therapy. Alternatively, his dentist may have made a misdiagnosis and incorrectly expected the canines to spontaneously align to class I. Although the explanation is unknown, it is evident that the net result has been anchorage loss. The extraction spaces are closed and no residual space is left to complete alignment of the upper lateral incisor or overjet reduction. C The principal problems here are an increased overjet and mild labial segment crowding. Although the canine relationship is ½ unit class II, this does not represent an underlying skeletal problem, but reflects the anchorage loss and crowding. A functional appliance is unsuitable as the patient has a class I skeletal relationship, class I molar relationship, and the upper incisors do not appear to be proclined. He needs fixed appliance therapy to detail his occlusion. Space is required to align the incisors and reduce the overjet. However, this cannot be created through dental extractions as he has already lost four premolars. So, he needs to be managed with distalization of the upper molars using headgear or an alternative method. In this case, the start molar relationship is class I. Therefore, the final molar relationship will be slightly class III. This will create sufficient space for anterior alignment. Although headgear is not a popular choice, particularly with a possible history of poor co-operation, in this instance it was successful.

Answer 8.4 A This is a facemask, commonly known as the Delaire facemask. It is also called protraction headgear. B It is used to apply traction to the maxilla. It is useful in class III cases and in patients with clefts where the maxilla is usually retrusive. It has been helpful in hypodontia cases when the posterior teeth need to be moved mesially but there is concern that the reciprocal effect on the anterior teeth would create a reverse overjet. This is an example of when anchorage needs to be lost rather than enhanced, and the facemask can achieve this. However, this technique may be succeeded by temporary orthodontic implants. C The appliance depends completely on patient compliance. The brow pad and chin cup are uncomfortable to wear, particularly in warm weather, and excessive sweating underneath

154 

Illustrated Questions in Orthodontics

these can lead to a skin rash. Cotton linings may help overcome this or, alternatively, perforations in these areas can be introduced. Traction is usually by elastics and, if these chafe at the angle of the mouth, significant discomfort can arise. Furthermore, saliva passing down the elastics can exacerbate this problem. D Elastics attached to hooks on the facemask are used to apply traction either to a removable appliance or fixed appliance. Such traction also keeps the facemask in place on the chin and forehead. The elastics should be attached to the middle of the frame to prevent them chafing at the angle of the mouth.

Answer 8.5 A This patient has an overerupted upper right second premolar, due to the opposing lower right second deciduous molar being retained and infraoccluded. The lower right second premolar is developmentally absent. An implant-retained restoration is ultimately planned to replace it, but the upper right second premolar needs to be intruded first. A temporary orthodontic implant has been positioned both in the buccal and palatal alveolus, and elastomeric traction has been placed between the tooth and the implants. These mechanics have been supported by a sectional archwire. B The orthodontic implant must be placed in attached mucosa, avoiding adjacent structures such as tooth roots and unerupted teeth. C The implant can be loaded immediately after placement. D Orthodontic implants can be used to support anchorage, to apply directional force to impacted teeth, and to distalize and intrude/extrude teeth.

Answer 8.6 A This patient has two impacted teeth, namely the upper right lateral incisor and the upper right permanent canine. B A temporary orthodontic implant has been placed in the buccal alveolus and the upper right canine has been bonded with a gold chain. Elastic traction has been applied to move the crown of the upper right canine distally, in the hope this will allow the upper right lateral incisor to erupt.

chapter nine Problems Answer 9. A Enamel decalcification has occurred during treatment with fixed appliances. This has progressed to cavitation on the upper right lateral incisor and adjacent canine. B The aetiological factors are the same as for any form of dental caries, i.e. substrate (diet high in sugar), susceptible tooth surfaces, and plaque. Carbonated sugar drinks are frequently implicated. Frequent consumption of a high-energy drink was responsible in this instance. C Unfortunately, such iatrogenic damage is common. More than 50% of patients have at least one white spot after completion of orthodontic treatment. D Decalcification can be avoided in patients who limit their sugar intake (no carbonated drinks or sweet snacks, including sweet drinks, between meals). Good oral hygiene and fluoride toothpaste are essential. Daily use of a fluoride mouth rinse at an alternative time than using fluoride toothpaste is also of proven benefit. Patients should continue with routine dental examinations with their general dental practitioner during orthodontic treatment. E Mild decalcification may improve post treatment with subsequent microscopic enamel surface loss. Microabrasion may be applied to moderate surface lesions. Severe cases may need restoration. Cavities should be managed appropriately, according to their site and severity, and should not be neglected during orthodontic treatment.

Answer 9.2 A This patient has a moderately crowded, mild class III incisor relationship with an intruded upper right central incisor. The intruded incisor is slightly grey in colour compared to the adjacent teeth, suggesting that it is non-vital and space loss has occurred. These signs indicate that this tooth position is a result of an old intrusive traumatic injury occurring subsequent to tooth eruption. Additionally, there is localized marginal gingivitis. B Space has been created to accommodate the central incisor. The dental arches are now in large rectangular steel wires. These are of sufficient rigidity to enable a light, flexible wire (in this instance, a nickel titanium wire) to be ‘piggy backed’ to extrude the intruded tooth. However, the adjacent teeth have become slightly intruded rather than the central incisor becoming extruded, as seen by the loss of the overbite. This suggests that the intruded incisor is ankylosed. C Permanent teeth with complete root formation that are traumatically intruded can become rapidly ankylosed, even within a week. Therefore, they should be extruded orthodontically as soon as possible after the injury. Permanent teeth with immature roots have greater chance of spontaneous eruption and do not require immediate treatment. However, they should be kept under close review and treated quickly if signs of devitalization and ankylosis are seen.

Problems  157

Answer 9.3 A This is a view of the lingual surfaces of the lower incisors, demonstrating generalized erythema and localized gingival recession. B The gingival recession is most likely to be traumatic in origin, compounded by the presence of plaque. In this case, the lingual flange of a medium opening activator functional appliance caused the damage. C The source of the trauma must be established and the mucosa must be given opportunity to heal. In this instance, the acrylic lingual flange was relieved over the affected area and the patient was advised to leave the appliance out for a few days. Oral hygiene instruction was given.

Answer 9.4 A The roots of the upper incisors have undergone extreme resorption during orthodontic treatment. B Yes. Root resorption is inevitable to an unpredictable degree during treatment with fixed appliances, although such extreme resorption as demonstrated here is uncommon. However, it rarely compromises the longevity of the teeth. C Many risk factors have been associated with this problem:

Root form Blunt, pipette-shaped, or short roots are thought to be the most susceptible to root resorption.

Ectopic canines Palatally impacted canines are associated with an increased risk of root resorption of the adjacent incisor and premolar teeth. This may be due to increased treatment duration.

Trauma Roots already showing signs of resorption due to trauma prior to orthodontic treatment will have increased risk of resorption. Traumatized teeth without root resorption are at no greater risk than non-traumatized teeth.

Treatment mechanics Fixed appliances, rectangular wires, class II traction, tooth intrusion, and the movement of root apices against cortical bone are all associated with root resorption.

Age Patients with complete root formation are at greater risk of root resorption than those with immature root apices.

Gender Females are more susceptible to root resorption than males.

Length of treatment Longer active treatment is thought to be more damaging than short courses of treatment, although this is equivocal.

158 

Illustrated Questions in Orthodontics

Individual susceptibility Patients with pretreatment resorption will have increased root resorption during orthodontic treatment.

Systemic problems Endocrine disturbances such as hyperparathyroidism and hyperthyroidism may increase susceptiblity to root resorption. D This problem can be minimized by: i taking a careful history and examination, including radiographs, before treatment commences, enabling the risk factors to be assessed. If the upper incisor roots have suspicious anatomy, long cone periapical radiographs should be taken as a baseline record. Cone beam tomography offers a three-dimensional perspective which is also useful. ii use of light forces and gentle mechanics iii minimizing the length of treatment iv monitoring progress in susceptible patients. Cases with signs of root resorption six months into treatment are likely to have severe root resorption at the end of treatment. Therefore, revise treatment aims and consider stopping treatment prematurely.

General feedback Consider the risk:benefit ratio carefully before treatment and warn all patients of the possibility of excessive root resorption occurring.

Answer 9.5 A This hyperkeratosis and ulceration of the buccal mucosa is caused by chronic irritation from an overextended archwire. B The archwire needs to be shortened and carefully tucked in distally to the last molar band. The patient needs to be given wax, when the appliance is first fitted, as a temporary solution to irritation from sharp appliance attachments.

Answer 9.6 A B

This is oral candidosis related to wearing an upper removable appliance. This problem needs to be managed by: i the institution of meticulous oral hygiene ii topical antifungal agents iii soaking the appliance in proprietary cleaning solutions iv reassessment of the aims of treatment and consideration of an alternative approach (e.g. suck-down style retainer which does not have palatal coverage) or cessation of treatment.

Answer 9.7 A The buccal mucosa bears the imprint of a reverse loop Hawley labial bow, caused by a badly fitting upper removable appliance. B The appliance needs to be remade more competently, with careful consideration given to modifying the design so that the wirework is more closely contoured to the teeth.

Problems  159

Answer 9.8 A A quadhelix appliance has caused this imprint on the dorsal aspect of the tongue. B The quadhelix must be examined to ensure that it contours the palate closely without palatal irritation and the patient needs simple reassurance that the imprint will resolve when the quadhelix is removed.

chapter ten Retention Answer 0. A These are removable upper and lower Hawley retainers. The lower has additional labial acrylic to maintain tooth alignment. B They are used to maintain tooth position after orthodontic treatment. Teeth will quickly relapse towards their initial position due to the recoil of the periodontal fibres, if not held in position until fibre remodelling is complete, and even then will still be at risk of becoming irregular over time. Individual orthodontists have their preferred regime:  full-time wear for 3 months followed by night-time wear for 9 months, missing occasional nights towards the end of the 2-month period is quite common. Patients can be advised that the longer they wear their retainers, the longer their teeth will stay straight. C Unfortunately, relapse to an unpredictable degree is common after orthodontic treatment and 75% of cases will have a recurrence of lower incisor crowding. Factors contributing to relapse are: unfavourable soft tissue relationship, such as incompetent lips and tongue thrust; unfavourable skeletal growth; periodontal fibres remaining under tension post treatment; and inappropriate tooth movements which have positioned the teeth out of soft tissue balance. D Spaces, such as diastemas, and rotated teeth are particularly susceptible to relapse. Rotated teeth can be overcorrected to aid stability and the periodontal fibres can be severed electively after derotation (a procedure known as pericision or circumferential fibrotomy). Retention (with fixed or removable retainers) should be long term for those who have unstable features of their malocclusion, pretreatment. Anterior open bites and deep bites are prone to relapse, as are cases where tooth movement has been performed in order to compensate for severe skeletal discrepancies.

Answer 0.2 A This is a vacuum-formed retainer made from thermally moulded plastic. B This type of retainer is aesthetic and comfortable, lacking palatal coverage, and cheap to make. The retainer is accurately contoured over the individual teeth positions, which is a benefit if the teeth are well positioned but a disadvantage if any settling is required. This retainer is relatively flimsy compared to an acrylic and wire retainer and is unlikely to maintain upper arch expansion as effectively due to the lack of palatal coverage. Finally, patients must be instructed not to eat or drink anything other than water whilst wearing this retainer, as it acts as a reservoir to hold liquids against enamel (like a bleaching tray), which can cause caries.

Answer 0.3 A This is a fixed retainer.

Retention  161 B It is used to maintain alignment of the teeth post treatment. Clinical indications for use include spacing, (e.g. diastema, periodontal drifting), severely rotated incisors, and situations where long-term retention is required. C A common problem in use is that the retainer may become detached from one tooth, which could predispose to tooth decay or relapse if unnoticed. Interdental cleaning is difficult but possible with persistence. Bonding a fixed retainer can be time consuming. Although it is possible to use fixed retainers for the upper incisors, sometimes an increased overbite at the end of treatment can prohibit the use of a fixed retainer in the upper arch because the lower incisors will occlude into the retainer. This causes bite disturbance and increases the likelihood that the fixed retainer will fail. D An impression of the dentition is taken at the visit prior to debond and a working model constructed. A multistrand wire is shaped, ideally in the laboratory prior to the debond appointment or at the chair side, to fit the lingual tooth surfaces in a position where it will be out of occlusion. Following debond, the lingual tooth surfaces are etched and bonded with composite cement into which the retainer is placed before light curing the adhesive. A custom-made jig supporting the retainer is a useful aid to hold the retainer in place whilst the composite sets. E Other options to a fixed retainer include long-term retention with a removable appliance, provided that the patient is co-operative. Proprietary fixed retainers are available commercially or bonding tape/chain can be used from a reel. Alternatively, a lingual bar supported by bands on the canines could be used. F This patient has two developmentally absent lower incisors.

Answer 0.4 A This photograph demonstrates two styles of retainer, namely a fixed retainer that is attached between the premolars, supplemented by a removable vacuum-formed retainer that is worn at night-time only. B The advantage of using both types of retainer simultaneously is that the anterior teeth are retained full time indefinitely, whilst the posterior teeth are retained by the occlusion and the removable retainer. Removable retainers are often lost or forgotten and can be inconvenient to wear long term. However, fixed retainers can become detached from the teeth without the patient realising, so use of the removable retainer helps minimize this risk. There are additional costs in terms of laboratory bills and chair time in providing two styles of retainer. C The fixed retainer is attached to the premolars as a means of minimizing the risk of the first premolar extraction space reopening.

Index Note to index: page numbers in italic refer to figures.

A acrylic tooth, bonding  7–8 Active—Retentive components—Anchorage— Baseplate (ARAB)  35 Adams cribs  66, 7, 36, 42 advancement genioplasty  0 alveolar bone grafting  29 ANB value  5, 42, 04, 07, 0, , 26 Eastman analysis  5, 04 anchorage  84–9, 52–5 loss  36, 53 Andrews’ class I molar ­relationship  2, 04 Andrews’ six keys to occlusion 2, 04 Angle’s class I molar relationship 2, 86, 04, 53 see also buccal segment relationship ankylosis  22, 8, 56 anterior crossbite  24, 7 anterior open bite  0, 09, 2–3 appliances fixed  72–83, 5, 46–5 functional  66–7, 42–5 removable  58–65, 08, 6, 7, 34–9 arch expansion  60, 35 archwires  24, 35, 43–9, 54, 58 with closing T loops  77, 48–9 ideal 49 asymmetrical eruption  5, 3

B backward mandibular growth rotation 23 balancing extraction  25 bands 46 see also brackets Begg appliance  46

bimaxillary osteotomy  09 bimaxillary proclination  9, 4 bone grafting  29 bone resorption  57, 33 brackets  46, 47 ceramic  73, 47 Damon 47 Innovation R brackets  75, 47 lingual  76, 48 lingual vs labial  47 self-ligating  75, 47–8 Smartclip 47 bridgework, conventional  8 buccal segment distalization 36, 53 buccal segment interdigitation 25 buccal segment relationship  8, 86, 08, 09, 53 class I  5, 2 class II  8, 08, 52 class III  7, 3 result of treatment  24

C candidosis  95, 58 canine guidance during lateral mandibular excursion  2, 04 canines aesthetic modification  20 early loss of deciduous  25 ectopic 57 palatally impacted  9, 2, 4 treatment  2, 5 palatally positioned  25 palpability and eruption, age at 4 position of the upper permanent  39, 25 root movement  6, 08 Caucasian population, missing teeth 20 centreline 25 upper shift  25 ceramic brackets  73, 47

Clark’s Twin Block appliance 67, 42 cleft lip and palate 46–8, 28–30 abnormalities in dentition  30 aetiology 28 classification 28–9 incidence 28 preservation of teeth  30 surgical management  29–30 Clincheck 39 clip-over clasps  37 clip-over Twin Block  69, 43 Coffin spring  49 compensating extraction  25 continuous positive airway pressure (CPAP)  40 cranial base angle  0 crossbite  23, 6 anterior  24, 7 crossbite correction, unilateral screw plate  36 crowding causes  37, 24 early loss of deciduous teeth 25 extent of  37, 24 relief of  6, 07 curve of Spee  2, 04, 24

D decalcification  90, 56 deciduous teeth, early loss  25 Delaire facemask  87, 53 dental caries  56 dental centrelines  06 dental extractions, extraoral and intraoral factors  24 dental panoramic tomogram (DPT) 06 dentigerous cyst  5, 20, 3 dentoalveolar change (teeth tip) 42 dentoalveolar compensation  42, 43, 26, 27 devitalization 56

164 Index diastema  23, 6, 30 opening 50 digit habit  see thumb sucking distalization of molars  36, 53 dummy use  6 dynamic occlusion  3 group function  05 mutually protected occlusion (MPO) 05

E Eastman analysis, ANB value 5, 04 elastic wear, class II traction 46, 47 enamel decalcification  90, 56 enamel reduction  07 eruption patterns, asymmetrical 5, 3 examination and diagnosis  4–33, 06–2 adjunctive procedures  06 extraoral/intraoral 06 expansion screw  60, 35 extraction balancing 25 compensating 25 extraoral traction (EOT) tubes 43 extrusion of teeth  9, 56

F facemask  87, 53 facial asymmetry  8, 3 facial growth, excessive anterior 3 facial vertical dimensions  5, 04 facial thirds  07 fixed appliances  72–83, 5, 46–5 archwire with closing T loops 77, 48–9 bonding, direct/indirect  48 Incognito appliance  76, 48 nickel titanium molar ­derotator  8, 50 quadhelix  79, 49 standard edgewise system 46 straightwire appliance  72, 46 Tip Edge appliance  73, 46–7 upper with ceramic brackets 73, 47 fixed retainer  00, 60– fraenal attachment  23, 49, 6, 30– Frankel functional appliance 68, 43

Frankfort–mandibular planesa angle  5, 8, 2, 04, 08, 09, 0, 42 increase  7, 2, 3 functional appliances  66–7, 42–5 Clark’s Twin Block  67, 42 clip-over Twin Block  69, 43 Frankel functional  68, 43 Herbst  70, 44 medium opening activator (MOA)  66, 42 fused teeth  32

G gemination  53, 3–2 gingival recession  92, 57 gingivitis  9, 56 glass ionomer, bond strength  46 gonial angle, reduced  

H Hawley retainers  96, 98, 58, 60 hayrake appliance  6, 6, 36 nonremovable  82, 50 headgear  85, 52–3 extraoral traction (EOT) tubes 4 J hook or cervical pull  24 for lingual arch.  52 protraction 52 safety 53 Herbst appliance  70, 44 histology  55–7, 32 bone resorption  57, 33 bone-forming activity  55, 32 compression of periodontal ligament  56, 32–3 Howship’s lacunae  57, 33 hyalinization  56, 32–3 hyalinization  56, 32–3 hyperkeratosis  94, 58 hypodontia  29, 20 hypomineralization (MIH)  54, 32 hyrax screw  49

I impaction canines  9, 2, 89, 4, 54–5 incisors  89, 54–5 treatment options  9 vs loss  25, 28, 7, 9 implants single 8 temporary  88, 89, 09, 54 imprint  97, 59 incisor relationship cingulum plateau  2, 04

classes  2, 04 Class I  6 class II division I  7, 8, , 43, 67, 08, 27, 42 class II division II  3, 0, , 25 Class III  8, 9, 07, 56 dentoalveolar compensation 3 overjet  2, 9, 04, 09, 0 incisors avulsion 7 geminated  53, 3 inclination 25 interincisal angle  3,  missing  29, 20 palatal cingulum  23 proclination  6, 08, 3, 7 retroclination  3, , 7 talon cusp  52, 3 trapping  IncognitoTM appliance  76, 48 Index of Treatment Need (IOTN) 27 Innovation R brackets  75, 47 interincisal angle  24 Invisalign® appliance  63, 39

J Japanese population, missing teeth 20 jaws, rotation during growth 23

L labiomental fold   lateral (posterior) open bite 22, 6 lingual arch  84, 52 lip, upper incisor/upper lip relationship 06 lip bumper  52 lip trapping/ lip incompetence  7, 9, , 08, 0

M malocclusion mild, treatment  39 see also incisor relationship mandible asymmetry  8, 3 large/small 27 mandibular condyles, seating  32, 04, 2 mandibular displacement  6, 08, 6 advancement 0 on closure  8, 3 rotation backwards  5, 3

Index  165 mandibular growth rotation, backward 23 mandibular retrognathia  7, 8, 2, 3, 43, 08, 09, 0, , 27 masticatory function  23 maxillary expansion  6, 08 rapid maxillary expansion (RME)  80, 49 maxillary impaction  0 maxillary proclination  9, 4 maxillary prognathism  2, 08, 0 maxillary teeth, transpositions 25, 7 maxillary–mandibular planes angle see Frankfort–mandibular planes angle medium opening activator (MOA)  66, 42, 57 menton 07 mesiobuccal cusp and groove 2, 04 midline supernumerary tooth 50, 3 missing teeth Caucasian population  20 hypodontia  29, 20 incidence 20 incisors  29, 30, 20 molar derotator  8, 50 nickel titanium  8, 50 molar incisor hypomineralization (MIH)  54, 32 molar relationships Angle’s class I  2, 86, 04, 53 see also buccal segment relationship molars deciduous, infraoccluded 27, 8 distalization with URA  36, 53 first, ideal time for extraction 54, 32 rotation  8, 50 second deciduous, early unilateral loss  25 mutually protected occlusion (MPO)  2, 04

anterior asymmetric  23, 6 posterior/lateral  22, 6 orthodontic study models  07 osteoclasts, origin  33 osteotomy, bimaxillary  09 overbite  3, 3, 32, 08, 0, , 2 and appliance retention  35 normal function  23 reduction  37, 23, 24, 42 traumatic  30, 3, 2 overjet  2, 9, , 30, 3, 04, 09, 0 increased  32, 86, 2, 5 reduced  7, 9, 32, 08, 09, 0, 2, 23, 42 reversed  42, 27, 53 overjet measurement, ­monitoring anchorage management 36

N

radiation dosage  26 radiography 06–7 cone beam tomography 06, 9 DPT 06 positioning 26 rapid maxillary expansion (RME)  80, 49 rare earth screens  26 removable appliances  58–65, 08, 6, 7, 34–40

nasolabial angle  24 nickel titanium molar derotator 8, 50

O obstructive sleep apnoea  40 occlusion, ideal  2–3, 04–5 open bite anterior  0, 27, 2–3, 8

P palatal finger springs  36 palatal impaction of canines  9, 2, 4 palatal shelf elevation  28 pathology  46–57, 28–33 Peer Assessment Rating (PAR) 27 periodontal ligament, compression  56, 32–3 plaque, and traumatic overbite 30, 3, 2 posterior bite capping  60, 35 premolars extractions, for bimaxillary proclination  9, 4 overerupted  88, 54 width  37, 24 problems  90–7, 56–9 protraction headgear  52 pseudotransposition 7

Q quadhelix  79, 49 imprint  97, 59

R

advantages/disadvantages 34 force delivered  34 Invisalign® 39 SomnowellTM appliance  40 upper (URAs), designs  36 uses 34–5 retainers  98–0, 35, 60– fixed  00, 0, 60– Hawley  96, 98, 58, 60 vacuum-formed retainer  99, 0, 60 root resorption  93, 57

S scissorsbite  0,  skeletal relationship anteroposterior discrepancy 27 camouflage by tooth position 3 Class I  3, 07 Class II  9, , 2, 09, 0 Class III  6, 7, 42, 2, 3, 26 dentoalveolar compensation 42, 27 vertical skeletal dimension and degree of overbite  23 see also buccal segment relationship ‘SLOB’ (same lingual, opposite buccal) rule, tooth position 4 soft tissue pogonion  , 0 Somnowell TM appliance  64 space creation  25 space discrepancy  37, 24 mild vs severe  37, 24 space maintenance  7, 20 guidelines 26 URA 36 space opening  8, 9, 20, 48, 6 spoon denture  7 springs, T and Z  35 straightwire appliance  72, 46 subnasale, defined  07 supernumerary tooth, midline 50, 3

T T loops  48–9 T springs  35 talon cusp  52, 3 temporary anchorage devices (TADs) 52 temporary orthodontic implants  88, 09, 54

166 Index thumb sucking  7, 23, 08, 4, 6, 25 hayrake appliance  6, 6, 36 nonremovable  82, 50 ‘Thumbguard’ 6 Tip Edge fixed appliance  73, 46–7 tongue pressure  6 tooth crowding see crowding; space tongue pressure tooth extrusion  23 tooth movement methods  46 tooth position  52 ‘SLOB’ (same lingual, opposite buccal) rule  4 torquing type spurs  37

traction, class II avoidance 23 elastic wear  46, 47 transpalatal arch  09 transpositions  25, 7 frequency 7 treatment planning  36–45, 22–7 aims 23 Index of Treatment Need (IOTN) 27 mandibular intercanine and intermolar widths  25 starting point  25 Twin Block appliance  67, 42 clip-over  69, 43

U ulceration of buccal mucosa 96, 58 upper arch expansion  60, 35 narrowing, thumb sucking 7, 08, 4 upper removable appliance (URA)  08, 6, 7

W wax placement  48 wire see archwires